SlideShare une entreprise Scribd logo
1  sur  30
Télécharger pour lire hors ligne
w
w
w
.exam
race.com
INSTRUCTIONS
Beforethetest:
1. DO NOT REMOVE THE SEALS OFTHE I>LASTIC ENVELOI>E OF THIS BOOKLET UNTILTHE SIGNALTO
START IS GIVEN.
2. Keep onlythe Admit Card, pencil, eraser and sharpenerwith you. DO NOT KEEl> with you books,
rulers, slide rulers, drawing instruments, calculators (including watch calculators), pagers, cellular
phones, stop watches or any other device or loose paper. These should be left at a place~ind icated by
the invigilator.
3. Use only HB pencil to fill in the Answer sheet.
4. Enter in your Answer Sheet: (a) in Box 3,t Test Form Number that appears at the b( Stt his p ge,
(b) in Box4, the Test Booklet Serial Numberthat appears atthe top ofthis page.
s. Ensure that your personal data have been entered correctly on Side- II of th6 sh .
6. Ensure that you have entered your 7-digit Test Registration Number in Box 2 fthe nswer sheet
correctly.
Atthe start oftheTest: 0 +
1. As soon as the signal to start is given, open the Test Booklet.r ~
2. ThisTest Booklet contains 36 pages, includingthe blan~"es~ately after opening the Test
Booklet, verify that all the pages are printed properly d · order. If there is a pnoblem with your
Test Booklet, immediately inform the invigiIator. ~Wi1 e rovid ed with a replacement.
How to answer: ~
1.
2.
3.
4.
nsofthe 1nv1g1lator. Cand1datesfound v1olatmg the 1nstruwonsWill be
2. You may retain this Test Booklet with you.
Candidares giving assisrance orseeking/receivingfrom anysource in answering quesrions or copying in any
manner in rhe resr willforfeir rheir chances of being consideredfor admission. Such candidares willforfeir rhe
righ r ro rhe scorecard. The resring aurhority reserves rherighr ro exdude any quesrion or quesrionsfrom rhis
Tesr Booklerforfinal examinarion.
w
w
w
.exam
race.com
Test Form Number:
SECTION I
1. A shop stores x kg of rice. The first customer buys half this amount pius half a kg of rice. The second
customer buys half the remaining amount plus half a kg of rice. Then the third customer also buys haIf the
remaining amount plus half a kg of rice. Thereafter, no rice is Ieft in the shop. Which of the following best
describes the value of x?
(1) Hx~6 (2)5~x~8 (3)9~x~12 (4)11~x~14 (5) 1Hx~18
Directions for Questions 2 and 3:
Let J(x) = a)+ bx + c, where a, b and c are certain constants and a " 0. It is known
that 3 is a root of f(x) = 0.
2. What is the other root ofJ(x) = 0?
(1) -7 (2) -4 (3) 2 (4) 6
3. What 1sthe value of a+ b+ c? ~
(1) 9 (2) 14 (3) 13 (4) 37 j"'Vbedeterm1ned
IS
(1) 78 (2) 19 (3) 20 f'l:J- 77
0~
~·
(5) 22
w
w
w
.exam
race.com
The figure below shows the pian of a town. The streets are at right angles to each other. A rectangular
park (P) is situated inside the town with a diagonal road running through it. There is also a prohibited
region (D) in the town.
('
BCJ
'· NoO•m "'~h~b'"''' from h~how• • '" h~'"'" <,L::,~:,.OnooIt•
number of poss1ble shortest paths that she can choose 1s ~- "XJ
(1) 60 (2) 75 (3) 45 (4) 90 ~U'
6. Neelam ndes her b1cycle from her house at -~atc; v1a B takmg the shortest path. Then the
number of poss1ble shortest paths that sh~~~lub
(I)Um (>) "" (>~()(.) """ 0) '"
Is dievalueof -
(q 0 ~ ~ "' i (<! 1 "' ........._
~""I, >, .... " •~~""'"'blookbo.rn Tho folio~"''""""' "lh~~'"""'" "•"
In each repetition, any two numbers, say a and b, currently on the blackboard are erased and a new
number a+ b- 1 is written. What wiII be the number left on the board at the end?
(1) 820 (2) 821 (3) 781 (4) 819 (5) 780
w
w
w
.exam
race.com
= seed(s(n)), otherwise,
wheres(n) indicates the sum of digits of n. For example,
seed(7) = 7, seed(248)= seed(2 + 4 + 8)= seed(14)= seed(1 + 4) = seed(5)= 5 etc.
How many positive integers n, such that n < 500, wiII have seed(n) = 9?
(1) 39 (2) 72 (3) 81 (4) 108 (5) 55
10. In a triangle ABc; the lengths of the sides AB and AC equal17.5 em and 9 em respectively. Le
point on the line segment BC such that 1>0 is perpendicular to BC If 1>0 = 3 em, then what is th
em) ofthe circle circumscribingthetriangle ABC?
(1) 17.05 (2) 27.85 (3) 22.45 (4) 32.25 (5) 26.25
11. What arethe last two digits of 7"SJos?
(1) 21 (2) 61 (3)01 (4)41 (5)81
•12. If the roots of the equation x'- a)+bx- c =0 are three consecutiv en
what is the smallest possible value of b?
many
5 F and H. Consider points P and Q, on L and inside ABCD, such that the angles APD and
20•. What is the ratio of the area ofABQCDP to the remaining area inside ABCD?
(2)2+1/3 (3} 1fH</3
9
l
(4)1+43 (5)243-1
17. Three consecutive positive integers are raised to the first, second and third powers respectively and
then added. The sum so obtained is a perfect square whose square root equals the total of the three
original integers. Which ofthe foil owing best describes the minimum, say m, of thesethree integers?
(1) Hm~3 (2) 4~m~6 (3) Hm~9 (4) 10~m~12 (5) 13~m~15
w
w
w
.exam
race.com
1 1
18.Find the sum 1+-+-+
12 22
1 1
1+2+1 +.............+
2 3
(1) :zoo&.~
1
(2) 2007--
2007
1 1
1+--+--
20072 20012
19. Two circles, both of radii 1 em, intersect such that the circumference of each or~ ugh the
centre of the circle ofthe other. What is the area (in sq em) of the intersecting regiv
(1).!!-./3 (2) 2n+./3 (3)4n-./3 (4)411+ ./3 ,cftJ•34 32 32 32 .:v
20. Rahim plans to drive from city A to station c, at these~mper hour, to catch a train arriving
there from B. He must reach Cat least 15 minutes b ""t arrival of thetrain. The train Ieaves B, located
500 km south of A at 8:00 am and travels at a ed km per hour. It is known that C is located
between west and northwest of B, with BC at 6 o so, Cis located between south and southwest of
A w1th AC at 30• to AB. The latest t1me by ~ must leave A and st1ll catch the tra1n 1s closest to
(1)6:15 am (2) 6:30am (3) ..!,.] (4) 7:00am (5) 7:15am
21. Cons1der a nght c1rcular cA~us4 em and he1ght 10 em. A cyhnder 1s to be piaced 1ns1dethe
oooo w<h ooo of''"i~'"fho "'"of fho oooo ''"' fho '"''" "'"'blo •~•I '""'~
area (in sq. em) of th cy der"+
00 ~ "' (3) ~ (4) ":" (S) '':"
~
w
w
w
.exam
race.com
Five horses, Red, White, Grey, Black and Spotted participated in a race. 1>s per the rules of the race, the
persons betting on the winning horse get four times the bet amaunt and those betting on the horse that
came in second get thrice the bet amaunt. Moreover, the bet amaunt is returned to those betting on the
horse that came in third, and the rest Iose the bet amount. Raju bets Rs. 3000, Rs. 2000 Rs. 1000 on Red,
White and Black horses respectively and ends up with no profit and no loss.
22. Which ofthe following cannot be true?
(1) At least two horses f1n1shed before Spotted
(2) Red f1n1shed last
(3) There were three horses between Black and Spotted
(S) Grey came In second r ~
23. Suppose, 1n add1t1on, 1t 1s known that Grey came 1n fourth. Then wh1ch of the f~ cannot be
true? ~.. +
(1) Spotted came in first CJ"'(2) Red finished last
((3
4
)) White came in second ~flj
Black came in second
(S) There was one horse between Black and White
0~
•
~
w
w
w
.exam
race.com
Marks (1) if Q can be answered from A alone but not from B alone.
Marks (2) if Q can be answered from B alone but not from A alone.
Marks (3) if Q can be answered from A alone as well as from B alone.
Marks (4) if Q can be answered from A and B together but not from any of them alone.
Marks (5) if Q cannot be answered even from A and B together.
In a single elimination tournament, any player is eliminated with a single loss. The tourname
multiple rounds subject to the foilowing rules:
(a) If the number of players, say n, in any round is even, then the players aree ro Q n/2 pairs.
The piayers in each pair play a match against each other and the winner mov on t ext round.
(b) If the number of piayers, say n, in any round is odd, then one of them is · bye, that is, he
automatically maves on to the next round. The remaining (n - 1) p3 are.f'ouped into (n - 1)/2
pairs. The piayers in each pair play a match against each other and n rs moves on to the next
round. No piayer gets morethan one bye inthe entire tourna'1~ :
Thus, if n is even, then n/2 players mave on to the next roun~~~d, then (n+1)/2 piayers mave
on to the next round. The process is continued till the fittr~~ , 1ch obviously is played between two
players. The winner in thefinal round is the champion f~u ment.
24. Q: What is the number of matches played '
w
w
w
.exam
race.com
SECTION II
This section contains 25 questions
Directions for Questions 26 to 28: Answer thefollowing questions based on the information given below:
For admission to various affiliated colleges, a university conducts a written test with four different
sections, each with a maximum of 50 marks. The following table gives the aggregate as welle.<::tth
sectional cut-off marks fixed by six different colleges affiliated to the university. A stud..._.......,
College I
College 3
College 4
College 6
S(~CtiOil ;
42
43
45
Se:tional Cut-off Marks
St~ction H St·ction C
42 42
45
176
175
171
45
180
44 176
26. Ad1tya d1d not get a call ~eve a smgle college. What could be the max1mum aggregate marks
obtained by h1m?
27. Bhama got c fr o! ges. What could be the minimum aggregate marks obtained by her?
(1)181 ~2)1 3)184 (4)196 (5)190
(1)180 )18 (3)196 (4)176 (5)184
(2) 21 (3) 25 (4) 35 (5) 41
w
w
w
.exam
race.com
The bar chart below shows the revenue received, in million US Dollars (USD), from subscribers to a
particular Internet service. The data covers the period 2003 to 2007 for the United States (US) and Europe.
The bar chart also shows the estimate revenues from the subscription to this service for the period 2008 to
2010.
.a
.~
"
1000
900
800
700
600
500
400
300
200
'i 100
;;
"'
0
JJ
03 04
l ........... t~.......
[
1
i.l[m ~..·.l····l·k·~·;,rl:.•.•.•.·..·..•.·. 11.·········1~ us CJ· ········· !! I[ ~~[ ~.Europe
tli ~,! i (!
t~.... ·r IIi Ii1
! .il .J![. J!l , ••
OS 06 07 08 09 10 CJ"'Vear
29. While the subscnpt1on 1n Europe has been grow1ng stl!d~sthat of the US, the growth rate 1n
Europe seems to be dechmng. Wh1ch of the follow ~~st to the percent change 1n growth rate of
2007 (over 2006) relative to the growth rate of 2 ver 04)?
(1) 17 (2) 20 (3) 35 (4) 60 (.,.,_.....
30. The difference between the estf f :·m ed ~tion in Europe in 2008 and what it would have been if it
were computed usingthe percenta ' rate of 2007 (over 2006), is closest to :
(1) 50 (2) 80 (3) 20 0 5) 0
31. In 2003, sixty percef. of subscribers in Europe were men. Given that woman subscribers increase at
the rate of 10 perc annt m and men at the rate of 5 percent per annum, what is the approximate
'~:llliilll~l:i ibers between 2003 and 2010 in Europe? The subscription prices are volatile
(4) 84 (5) 50
...-..... the annual percent change in the gap between subscription revenues in the US and Europe.
;m,._...,,t he year in which the absolute value ofthis change is the highest?
(1) 03-04 (2) 05-06 (3) 06-07 (4) 08-09 (5) 09-10
w
w
w
.exam
race.com
There are 100 employees in an organization across five departments. The following table gives the
departement-wise distribution of average age, average basic pay and allowances. The gross pay of an
employee is the sum of his/her basic pay and allowances.
lh·partrncnt'
fumbct- or Average ,vcrag{· Basic Allowances
~:mployees Age (Yl~an) !•ay (Rs. ) (% of Basic J•a..,
IIR 5 5000
()()!){)
Finance 30 6500
Bu~incs~ DeveliJf.}ll'lent 42
Maimcnancc iO J5
a department w1th h1gher average age to ~em 1th lower average age.
Questions below are independent : £f e 1-MJU
33. There was a mutual transfer of e between Market1ng and F1nance departments and transfer
of one empioyee from M arke~!'' 1>s a result, the average age of F1nance department 1ncreased by
one year and that of marke~rtm ent remained the same. What is the new average age of HR
department?
•(1) 30 (2) 35 (4) 45 (5) cannot be determined
transfer a - person with basic pay of Rs. 8000 from the Marketing department?
(1)~~)11% (3)13% (4)15% (5)17%
~mployees(each with a basic pay of Rs. 6000) are transferred from Maintenance department to
HR department and one person (with a basic pay of Rs. 8000) was transferred from Marketing department
to HR department, what will be the percemage change in average basic pay of HR departmem?
(1) 10.5% (2) 12.5% (3) 15% (4) 30% (5)40%
w
w
w
.exam
race.com
Abdul, Bikram and Chetan are three professianal traders who trade in shares of a company XYZ Ltd. Abdul
follows the strategy of buying at the opening of the day at 10 am and selling the whole Iot at the close of
the day at 3 pm. Bikram follows the strategy of buying at hourly intervals: 10 am, 11 am, 12 noon, 1 pm
and 2 pm, and sellingthe whole lot at the close of the day. Further, he buys an equal number of shares in
each purchase. Chetan follows a similar pattern as Bikram but his strategy is somewhat different. Chetan's
total investment amaunt is divided equally among his purchases. The profit or loss made by each investor
is the difference between the sale value at the close of the day less the investment in purchase. The
"returri' for each investor is defined as the ratio of the profit or loss to the investment amaunt e d
as a percentage.
36. on a "boom" day the price of xvz Ltd. keeps rising throughout the day and peaks 0day. Which trader got the minimum return on that day?
(1) Bikram (2) Chetan (3) Abdul (4) Abdul or Chetan (5) cannot be dG d
37. On a day of fluctuating market prices, the share price of XYZ Ltd. e~a Q in, i.e., it is higher at
the close of the day compared to the opening value. Which trader got th~m return on that day?
(1) Bikram (2) Chetan (3) Abdul (4) Bikram or Chetan G ot be determined
38. Which one ofthe following statements is always true? f'l:j-
(1) Abdul will not be the one with the minimum retur .t.,(2) Return for Chetan will be higherthan that of Bikr m ~
(3) Return for Bikram will be higherthan that of e
(4) Return for Chetan cannot be higher than tha Ab
(5) none of the above ~
One day, two other traders, Dane~n 11 oi tl Abdul, Bikram and Chetan for trading in the shares of
XYZ Ltd. Dane followed a strategy · qual numbers of shares at 10 am, 11 am and 12 noon, and
selling the same numbers at21, and 3 pm. Emily, on the other hand, followed the strategy of
buying shares using all her m a 0 am and selling all of them at 12 noon and again buyingthe shares
for all the money at 1 p and & · selling all of them at the close of the day at 3 pm. At the close of the
b ve~
(1) Share price was at its lowest at 2 pm
(2) Share price was at its lowest at 11 am
(3) Share price at 1 pm was higher than the share price at 2 pm
(4) Share price at 1 pm was higher than the share price at 12 noon
(5) none of the above
w
w
w
.exam
race.com
~u. ~ll ell ~ IJI n..~ VV d :> dl. n :> 111 ~11~:>1. en
(1) 10 am (2) 11 am (3) 12 noon (4) 1 pm (5) cannot be determined
w
w
w
.exam
race.com
(i) There arethree houses on each side ofthe road.
(ii) These six houses are labeled as P, Q, R, S, T and U.
(iii) The houses are of different colours, namely, Red, Blue, Green, Orange, Yellow and White.
(iv) The houses are of different heights.
(v) T, thetallest house, is exactIy opposite to the Red coloured house.
(vi) The shortest house is exactIy opposite to the Green coloured house.
(vii) U, the Orange coloured house, is located between P and S.
(viii)
(ix)
(x)
R, the Yellow coloured house, is exactly opposite toP.
Q, the Green coloured house, is exactIy opposite to U.
P, the White coloured house, is tallerthan R, but shorter than Sand Q.
41. What is the colour of the tallest house? r ~
(1) Red (2) Blue (3) Green (4)Yellow (S) none of these ~
42. What is the colour of the house diagonally opposite to theYellow col0 st
(1) White (2) Blue (3) Green (4) Red (S) none of these CJ
~'(j
43. Which is the second tallest house?
(1) p (2) s (3)Q (4) R
w
w
w
.exam
race.com
In a sports event, six teams (A, B, C, D, E and F) are competing against each other. Matches are scheduled
in two stages. Each team plays three matches in Stage-1 and two matches in Stage-11. No team plays against
the same team morethan once in the event. No ties are permitted in any of thematches. The observations
after the completion of Stage-I and Stage-II are as given below.
Stage-1:
• Oneteam won all the three matches.
• Two teams lost all thematches.
• D lost to Abut won against Cand F.
• E lost to Bbut won against Cand F.
• BIost at least one match.
• Fdid not play against thetop team of Stage-1.
Stage-11:
• The leader of Stage-llost the next two matches. ~ +
• Of the two teams at the bottom after Stage-1, oneteam wo at es, wh1le
the other Iost both matches.
• One moreteam lost both matches 1n Stage-11. f'l::f-
44. The team(s) w1th the most w1ns 1n the event 1s (ar · ~
(l)A (2)A&C (3)F (4)E &
(l)A D &F (3) E& F (4) D, E& F (S) D& F
w
w
w
.exam
race.com
Telecom operators get revenue from transfer of data and voice. Average revenue received from transfer of
each unit of data is known as ARDT. In the diagram below, the revenue received from data transfer as
percentage of total revenue received and the ARDT in US Dollars (USD) are given for various countries.
• •fJ-oi~flii'WI: {~:l,!:;t.~}
$5
e ' he same in both countries.
vent,_ in India is about 2times that of Singapore
nue in India is about 4times that of Singapore
revenue in Singapore is about 2 times that of India
(S) Total revenue in Singapore is about 4 times that of India
•Ja~n (lD13.10~)
UK
$Hi
(3) Germany and USA
w
w
w
.exam
race.com
I Ul I I IUIQ Ql IU UUUUI o; I Ul .JYY <;U<;I lo ............UI I I<; 0.1 IQO. II I ~ U.I.UJ 0.1 I<; O.UO.Q I I o; Y<;l IU<; II I I I IUIQ I'"' 0. YY 1'-'<; 0.1 IQO. Ul ,J YY<;U<;I I
and that the volume of data transfer is the same in both the countries. What is the percentage increase of
ARDT in India if there is no change in ARDT in Sweden?
(1) 400% (2) 550% (3) 800% (4) 950% (5) cannot be determined
w
w
w
.exam
race.com
This section contains 40 questions
Directions for questions 51 to 54: In each question, there are five sentences. Each sentence has a pair of
words that are itaIicized and highlighted. From the italicized and highlighted words, select the most
appropriate words (A or B)to form correct sentences. The sentences arefollowed by optionsthat
indicate the words, which may be selected to correctly compietethe set of sentences. From the options
g1ven, choose the mostappropriate one.
51.
An1ta wore a beaut1ful broach {A}/brooch{B} on the lapel of herJacket.
coundlfol(A}/counsefoi(B}. CJI wauld like youradvke(A}/advise{B} on which job I should choose.
The last scene provided a climactic(A)/climatic{B) ending to the film.
Jeans thatjiair{A}/ffore{B} at the bottom are in fashion these days. +
(1) BABM (2) BABAB (3) BAMB (4)ABABA
(1) BBABA (2) BBAM (5) BABAB
53.
ke+Nay as thetrain elimbed the hiII.
(3) BAABB (4)ABBM (5) BBBBA
The hockey team cauld not deal with his unsociable{A}/unsociai{B} tendencies.
(1) BABBA (2) BBABB (3)BABM (4)ABBAB (5)AABBA
w
w
w
.exam
race.com
usage (including spelling punctuation and logical consistency). Then, choose the most appropriate
option.
55.
A. In 1849, a poor Bavarian imigrant named Levi Strauss
B. landed in San Francisco, california,
C. at the invitation of his brother-in-law David Stern
D. owner of dry goods business.
E. Th1s dry goods busmess would later become known as Lev1 Strauss & Company.
(1) B only (2) Band C (3)AandB (4) A only (S)A, B andD
CJ
56.
A. In response to the allegations and condemnation pouring in,
B. Nike implemented comprehensive changes in their labour policy.
C. Perhaps sensingthe rising tide of global labour concerns, ~., +
D. from the public wauld become a prominent media issue, " - /
E. Nike sought to be a industry leader in employee relations. r ~
(1) D and E (2) D only (3)Aand E (4)Aand D~~dE
57. .t"'V
A. Charges and counter charges mean nothing ~
B. to the few miIlion who have lost their hom
D. is still unableto reach hundreds wh re o ed.
E. The death count haveJUSt be::.t..=__C
(1)Aonly (2)Conly ~~:v~ (4)A,CandD
58.
to ma~ r you.
(S) D only
B. Because you'd Inett I associate you more with my parents than with me.
·~~
C. And yet yo"'"""'-.., my cousins in Calcutta, who seem so innocent and obedient when I visited
them.
urious about me in the least.
u did make effort to meet me.
(2)AandB (3)AandE (4) D only (S)Aand D
w
w
w
.exam
race.com
Given below each question arefive pairs of words. Choosethe pairthat best compietes the sentence.
59.
The genocides in Bosnia and Rwanda, apart from being mis-described in the most sinister and
___ manner as 'ethnic cleansing', were also blamed, in further hand-washing rhetoric, on
something dark and interiorto _____ and perpetrators alike.
(1) 1nnovat1ve; commumcator
(2) enchant1ng; leaders
(3) d1s1ngenuous; v1ct1ms
(4) ex1gent; explo1ters
(5) trag1c; sufferers
contrary, anc1ent astronomers wereforced to _____ that certa1n bod1es ml~=~1n c1rcles about
po1nts, wh1ch 1n turn moved 1n c1rcles about the earth. ~
(1) scrut1n1zers; believe (4) observers; concede ~.. +
(2) observers; agree (5) students; con~~"'fJ
(3) scrut1n1zers; suggest .:'-/
61. £~Every human be1ng, after the fwst few days of h1s hfe, 1 :U'twofactors; on the one hand, there
:::'s-=--=--.___endowment; and on the~ ere 1s the effect of env1ronment, 1nclud1ng
(1) const1tut1onal; weather ~~~nom1c;learn1ng
(2) congenital; educat1on ;f )genet1c; pedagogy
(3) personal; climate
62. (kExhaustion of natural resou , d ruct1on of 1nd1v1dual 1n1t1at1ve by governments, control overmen's
mmd> by'"'"~ "'"""ood propogoodo •~ ~'"""h• '"'I"~"' wh'oh •PP~' <o
be on the 1ncreas ul~f the 1mpact of sc1ence upon m1nds su1ted by ____to an earher kmd
of world.
(1) tenet n (4) organs; trad1t1on
(2) a s· h1 1t1ons (5) departments; repuls1on
w
w
w
.exam
race.com
incorrect or inappropriate.
63. Run
(1) I must run fast to catch up with him.
(2) Our team scored a goal against the run of play.
(3) You can't run over him like that.
(4) The newly released book is enjoying a popular run.
(5) This film is a run-of-the-mill production.
64. Round
(1) The policefired a round of tear gas shells.
(2) The shop is located round the corner.
(3) Wetook a ride on the merry-go-round.
(4) The doctor is on a hospital round. ~• +
(5) I shall proceed further only after you come around to admitting it. " - /
65.Buckle CJ(1) .after the long hike our knees were beginning to buckle. ~
(2) The horse suddenly broke into a buckle. .t"'V
(3) The accused did not buckle under police interr ·~
(4) Sametimes, an earthquake can make a brid ,..... ckl .
(5) People should Iearn to buckle up as soon ey
66.File ~
(1) You will find the paper in the~~rV
(2) I need to file an insuran~· .
(3)The cadets were marchi i s gl file.
(4) File your na
1
i~~:~~~u ail polish.
(5) When the p~~n-+soldier brokethefile.
w
w
w
.exam
race.com
in the mostappropriate way.
67.
Most people at their first consultation take a furtive look at the surgeon's hands in the hope of
reassurance. Prospective patients look for delicacy, sensitivity, steadiness, perhaps unblemished pallor.
On this basis, Henry Perowne loses a number of cases each year. Generally, he knows it's about to
happen before the patient does: the downward glance repeated, the prepared questions beginning to
falter, the overemphat1c thanks dunng the retreat to the door.
(1) Other people do not commumcate due to thew poor observation.
(2)
(3) But Perowne h1mself 1s not concerned.
(4) But others will take the1r place, he thought. r ~
(5) These hands are steady enough, but they are large. ~
68.
Trade protectionism, disguised as concern for the elimate, is raisinl!l..liliw·~
concerns, powerful industrialized countries are holding out threat of a I
other.
(3) Jews in India were always tolerant.
(4) Reiigious tolerance has always been only afa9ade and nothing more.
(5) The pretty pastel streets are, thus, very popular with thetourists.
•ng competitiveness
n imports of energy·
w
w
w
.exam
race.com
70.
Given the cultural and intellectual interconnections, the question of what is 'Western' and what is
'Eastern' (or Indian) is often hard to decide, and the issue can be discussed only in more dialectical
terms. The diagnosis of athought as 'purely Western' or 'purely Indian' can be very illusory.
(1) Thoughts are not the kind of things that can be easily categorized.
(2) Though 'accidentalism' and 'orientalism' as dichotomous concepts have found many adherents.
(3) 'East is East and West is West' has been a discredited notion for a longtime now.
(4) Compartmentalizingthoughts is often desirable.
(5) The origin of athought is not the kind of thingto which 'purity' happens easily.
CJ
,Jo·
&'(j
0~
•
~
w
w
w
.exam
race.com
"-'IIUU;)t:: U It:: Ill U;)l dJ-'J-'1 Ut-JII dlt:: dll;) YYt::l lU t::d'-'11 yut::;)UUIIo
Language is not a cultural artifact that we learn the way we learn to tell time or how the federal
government works. Instead, it is a distinct piece of the biological makeup of our brains. Language is a
complex, specialized skill, which develops in the child spontaneously, without conscious effort or formal
instruction, is deployed without awareness of its underlying Iogic, is qualitatively the same in every
individual, and is distinct from more general abilities to process information or behave intelligently. For
these reasons same cognitive scientists have described language as a psychological faculty, a mental
organ, a neural system, and a computational module. But I prefer the admittedly quaint term "instinct."
It conveys the idea that people know how to talk in more or Iess the sensethat spiders know ho
webs. Web-spinning was not invented by same unsung spider genius and does not depend onlr:w~!l
the right education or on having an aptitude for architecture or the construction trades. Ra r, s rs
spin spider webs because they have spider brains, which give them the urge to spin aa etence
to succeed. Although there are differences between webs and words, I will enc age u to see
Ianguage in this way, for it helps to make sense of the phenomena we wiII explorr r ~
Thinking of language as an instinct inverts the popular wisdom, especially as it h~passed down in
the canon of the humanities and social sciences. Language is no more as i'Vntion than is upright
posture. It is not a manifestation of a general capacity to use -symbols: t ear old, we shall see, is a
grammatical genius, but is quite incompetent at the visual arts, a graphy, traffic signs, and
the other staples of the semiotics curriculum. Though language i ma ificent ability unique to Homo
sapiens among living species, it does not call for sequester~ ' of humans from the domain of
biology, for a magnificent ability unique to a particular i' ·_s is far from unique in the animal
kingdom. Same kinds of bats home in on flying inse usi pier sonar. Some kinds of migratory
birds navigate thousands of miles by calibrating t o ·ons of the constellations against the time of
day and year. In nature's talent show we are si y a e s of primate with our own act, a knack for
communicating information about who did w. t t ho by modulating the sounds we make when we
exhale
Once you begin to Ioak at language
adaption to communicate inforllloll~J!~
of thought, and, we shall se~a· · ot oreover, seeing Ianguage as one of nature's engineering marvels
- an organ with "that perf · ructure and co-adaption which justly excites our admiration, "in
Darwin's words- a ·spect for your ordinary Joe and the much-maligned English language
(or any language). of language, from the scientist's point of view, is part of our biological
.,."-iru> that parents teach their chiIdren or samething that must be elaborated in
~~e said, "Education is an admirable thing, but it is well to remember from time to
is worth knowing can be taught." A preschooler's tacit knowledge of grammar is
more'.ff't:q ~c:~!d than the thickest style manual or the most state-of-the-art computer language
same applies to all healthy human beings, even the notorious syntax-fracturing
athlete and the, you know, Iike, inarticulate teenage skateboarder. Finally, since Ianguage is
~~duct of a well-engineered biological instinct, we shall see that it is not nutty barrel of monkeys
that entertainer-columnists make it out to be.
71. Accordingto the passage, which ofthe following does not stem from popular wisdom on language?
(1) Language is a cultural artifact.
(2) Language is a cultural invention.
(3) Language is learnt as we grow.
(4) Language is unique to Homo sapiens
w
w
w
.exam
race.com
72. Which of the following can be used to repIacethe "spiders know how to spin webs" analogy as used
by the author?
(1) A kitten learningto jump over a wall
(2) Bees collecting nectar
(3) A donkey carrying a load
(4) A horse running a Derby
(5) A pet dog protecting its owner's property
73. Accordmgto the passage, wh1ch ofthe follow1ng 1s umque to human bemgs?
(1) Ability to use symbols while communicating with one another.
(2) Ability to communicate with each other through voice modulation.
(3) Ability to communicate informationto other members of the species.
CJ(4) Ability to use sound as means of communication.
(5) All ofthe above.
74. According to the passage, complexity of language cannot u0p!entsor at school to
ch1ldren because
(1) children 1nSt1nct1vely know language. flj(2) children learn the language on the1r own. ~
(3) language 1s not amenable to teachmg.
(4) ch1ldren know language better than the1r teac r ents.
(5) ch1ldren are born w1th the knowledge of s cs.
w
w
w
.exam
race.com
"-'IIUU;)t:: U It:: IIIU;)l dJ-'J-'1 Ut-JII dlt:: dll;) YYt::l lU t::d'-'11 yut::;)UUIIo
When I was little, children were bought two kinds of ice cream, sold from those white wagons with the
canopies made of silvery metal: either the two-cent cone or the four-cent ice cream pie. The two-cent
cone was very smaII, in fact it cauld fit comfortably into a child's hand, and it was made by taking the ice
cream from its container with a special scoop and piIing it on the cone. Granny always suggested I eat
only a part of the cone, then throw away the pointed end, because it had been touched by the vendor's
thing, the biscuit surfaces softening as they became soaked in creamy nectar.
give here: in theory the pies had been touched only by the machine; in practi
them against his hand while giving them to us, but it was impossible to i~e the. aminated area.
1was fascinated, however, by same ofmy peers, whose parents bas m t afour-cent pie but two
two-cent cones. These privileged children advanced proudly with ne c their right hand and one in
their left; and expertly moving their head from side to sidr & first one, then the other. This
Iiturgy seemed to me so sumptuously enviable, that man ed to be allowed to eelebrate it. In
vain. My elders were inflexible: a four-cent ice, yes; bu o t - nt ones, absolutely no.
assuming that in due course the tips of es ere discarded. The pathetic, and obviously
mendacious, justification was that a boy~- e WI h turning his eyes from one coneto the other was
more inelined to stumble over stones, e , acks in the pavement. I dimly sensed that there was
another secretjustification, cruelly p ag · , but I was unable to grasp it.
Today, citizen and victim of!a r society, a civilization of excess and waste (which the society of
the thirties was not), I reali t se dear and now departed elders were right. Two two-cent cones
instead of one at fou cents t signify squandering, economically speaking, but symbolically they
surely did. It was ·s pr. ise reason, that I yearned for them: because two ice creams suggested
·sely why they were denied me: because they Iooked indecent, an insult to
poverty, a di ;;,llllo;.'l....,.itious privilege, a boast of wealth. Only spoiled chiIdren ate two cones at once,
those chil9il,._m!o in fairy tales were rightly punished, as Pinocchio was when he rejected the skin and
h parents who encouraged this weakness, appropriate to Iittle parvenus, were bringing up
re 1n the foolish theater of "I'd like to but I can't." They were preparing them to turn up at
s cheek-in with a fake Gucci bag bought from a street peddler on the beach at Rimini
Nowadays the moralist risks seeming at odds with morality, in a world where the consumer civilization
now wants even adults to be spoiled, and promises them always something more, from the wristwatch
in the box of detergent to the bonus bangle sheathed, with the magazine it aceompanies, in a plastic
envelope. Like the parents of those ambidextrous gluttons I so envied, the consumer civiiization
pretends to give more, but actually gives, for four cents, what is worth four cents. You wiII throw away
the old transistar radio to purchase the new one, that boasts an alarm clock as well, but same
w
w
w
.exam
race.com
not Iast nearly so long as the glorious old Fiat 500, which, even when it broke down, could be started
again with a kick.
The morality of the old days made Spartans of us all, while today's morality wants all of us to be
Sybarites.
76. Which of the following cannot be inferred from the passage?
(1) Today's society is more extravagant than the society of the 1930s.
(2) The act of eatingtwo ice cream cones is akin to a ceremonial process.
(3) Elders rightly suggested that a boy turning eyes from one cone to the other was more li
(4) Despite seeming to promise more, the consumer civilization gives away exactlyO '
worth.
(5) The consumer civilization attempts to spoil chiIdren and adults alike. CJ
77. In the passage, the phrase "Iittle parvenus" refers to 0(1) naughty midgets. +
(2) old hags. CJ(3) arrogant people.
(4) young upstarts. ~
(5) foolish kids. "'v
(1) it made dietetic sense.
(2) it suggested intemperance. flj(3) it was more fun.
(4) it had a visual appeal.
(5) he was a glutton.
79. What does the author bl "r'iclW<Idays the moralist risks seeming at odds with morality"?
(1) The moralist of ves>ter·dav,_lllllll~·eccrme immoral today.
(3) dialectic.
(4) diatonic.
(5) diastolic.
1+s changed overthe years.
~,jO'Ci~~ with immorality have gone up.
of morality is fast becoming popular.
to the author, the justification for refusal to let him eat two cones was piausibly
w
w
w
.exam
race.com
"-'IIUU;)t:: U It:: IIIU;)l dJ-'J-'1 Ut-JII dlt:: dll;) YYt::l lU t::d'-'11 yut::;)UUIIo
A remarkable aspect of art of the present century is the range of concepts and ideologies which it
embodies. It is aimost tempting to see a pattern emerging within the art field - or alternative!y imposed
upon it a posteriori - similar to that which exists under the umbrella of science where the general term
covers a whole range of separate, though interconnecting activities. Any parallelism is however- in this
instance at least - misleading. A scientific discipiine develops systematically once its bare tenets have
been established, named and categorized as conventions. Many of the concepts of modern art, by
contrast, have resulted from the aimost accidental meetings of groups of talented individuals a'fi""J!
times and certain places. The ideas generated by these chance meetings had twofold conc."""''"
Firstly, a corpus of work wauld be produced which, in great part, remains as a concrete r rd he
events. Secondly, the ideas wauld themselves be disseminated through many di~ els of
communication - seeds that often bore fruit in contexts far removed from ta · ra · n. Not all
movements were exclusively concerned with innovation. Surrealism, for instan clai o embody a
kind of insight which can be present in the art of any period. This claim has bee " ally accepted so
that a sixteenth century painting by Spranger or a mysterious photogri ! t ~ can Iegitimately be
discussed in surrealist terms. Briefly, then, the concepts of modern re f many different (often
fundamentally different) kinds and resulted from the exposures fJ. ""rs, sculptors and thinkers
to the more complex phenomena of the twentieth century, i I o ever increasing knowledge of
the thought and products of earlier centuries. Different gr p a sts would collaborate in trying to
make sense of rapidly changing world of visual and spi~· I e er. nee. We should hardly be surprised if
no one group succeeded completely, but achiev e < through relative, have been considerable.
Landmarks have been established - concrete st me ' osition which give a pattern to a situation
which could easily have degenerated into al os. Beyond this, new language tools have been
created for those who follow - seml ] i ems wh1ch can prov1de a spnngboard for further
explorations.
The cod1fy1ng of art 1s often cnt1~ y one can understand that art1sts are wary of be1ng pigeon-
holed smce they are apt tar at mselves as 1nd1v1duals- somet1mes w1th good reason. The not1on
of self-express1on, howeve o e carnes qu1te the we1ght 1t once d1d; obJectiVIty has 1ts defenders.
There 1s good reason acce 1deas cod1f1ed by art1sts and cnt1cs, over the past s1xty years or so, as
st s ~ndependent existence - an independence which is not without its own
portant here. As an art movement slips into temporal perspective, it ceases
- becoming, rather, afossil. This is not to say it becomes useless or uninteresting
>'ili'"ollacan reconstruct the life of a prehistoric environment from the messages codified into
the sttl~~~~f fossil, so can an artist decipher whole webs of intellectual and creative possibility from
tructure of a 'dead' art movement. The artist can match the creative patterns crystallized
ructure against the potentials and possibilities of his own time. AS T.S Eliot observed, no one
nything from scratch; however consciously you may try to Iive in the present, you are still
involved with a nexus of behaviour patterns bequeathed from the past. The original and creative person
is not sameone who ignores these patterns, but someone who is able to translate and develop them so
that they confirm more exactly to his- and our- present needs.
w
w
w
.exam
race.com
(2)the dissemination of ideas through the state and its organizations.
(3)accidental interactions among people blessed with creative muse.
(4)patronage by the rich and powerful that supported art.
(5)systematic investigation, codification and conventions.
82. In the passage, the word 'fossil' can be interpreted as
(1) an art movement that has ceased to remain interesting or useful.
(2)an analogy from the physical world to indicate a historic art movement.
(3)an analogy from the physical worldto indicatethe barrenness of artistic creations in th
(4)an embedded codification of pre-historic life. e(5)an analogy from the physical world to indicatethe passing of an era associated wit an a
movement. r ~
83. In the passage, which of the following similarities between science and art~ad to erroneous
conclusions? ~.. +
(l)Both, in general, include a gamut of distinct but interconnect~n ~
(2)Both have movements not necessarily concerned with innov n.
(3)Both depend on collaborations between talented indivil l j
(4)Both involve abstract thought and dissemination o~f.·
(5)Both reflect compiex priorities of themodern worl~
84. The range of concepts and ideologies emb in art of the twentieth century is explained by
(l)the existence of movements such as surr
(2)1andmarks which give a pattern tot
only thrive in a 'greenhouse' insulated from the past biases.
s' and 'original thinking' interpret and develop on past thoughts to suit contemporary
w
w
w
.exam
race.com
"-'IIUU;)t:: U It:: IIIU;)l dJ-'J-'1 Ut-JII dlt:: dll;) YYt::l lU t::d'-'11 yut::;)UUIIo
To summarize the Classic Maya collapse, we can tentatively identify five strands. I acknowledge,
however, that Maya archaeologists still disagree vigorously among themselves-in part, because the
different strands evidently varied in importance among different parts of the Maya realm; because
detailed archaeological studies are available for only same Maya sites; and because it remains puzzling
why most of the Maya heartIand remained nearly empty of population and failed to recover after the
collapse and after re-growth of forests.
With those caveats, it appears to me that one strand consisted of population growth o
available resources: a dilemma similar to the one foreseen by Thomas Malthus in 1798 and
out today in Rwanda (Chapter 10), Haiti (Chapter 11), and elsewhere. 1>s the arch o
Webster succinctly puts it, "Too many farmers grew too many crops on too much
Compounding that mismatch between population and resources was the second
deforestation and hillside erosion, which caused a decrease in the amount of us
when more rather than less farmIand was needed, and possibly exacerbate by
drought resulting from deforestation, by soil nutrient depletion and other soiI p
struggleto prevent bracken ferns from overrunningthe fields. +
The third strand consisted of increased fighting, as more and mo pea ght over fewer resources.
Maya warfare, already endemic, peaked just beforethe coli~. 1 tis t surprising when one reflects
that at Ieast 5,000,000 people, perhaps many more, were o an area smaller than the state
of Colorado (104,000 square miles). That warfare w~ eased further the amount of Iand
available for agriculture, by creating no-man's lands ee ncipalities where it was now unsafe to
farm. Bringing matters to a head was the strand of t ange. The drought at the time of the Classic
collapse was not the first drought that the Ma ad ' through, but it was the most severe. tit the
time of previous droughts, there were still u ab1 parts of the Maya Iandscape, and people at a site
affected by drought could save thems~ oving to another site. However, by the time of the
Classic collapse the landscape was now II as no useful unoccupied land in the vicinity on which
to begin anew, and the whoIe po5 :ul ion u not be accommodated in the few areas that continued to
have reliable water supplies.
1>s our fifth strand, we ha..Qfln er why the kings and nobles failed to recognize and solve these
seemingly obvious problem~ining their society. Their attention was evidently focused on their
short-term concerns enricbing themselves, waging wars, erecting monuments, competing with each
other, and extra 1 ugfrfood from the peasants to support all those activities. Like most leaders
throughout h r , , the Maya kings and nobles did not heed long-term problems, insofar as they
perceived he II return to this theme in Chapter 14.
w stiII have same other past societies to consider in this book before we switch our
to e modern world, we must already be struck by some parallels between the Maya and the
ies discussed in Chapters 2-4. As on Easter Island, Mangareva, and among the Anasazi, Maya
• <it:tnmental and population problems led to increasing warfare and civil strife. As on Easter Island and
co Canyon, Maya peak population numbers were followed swiftIy by political and social coliapse.
ParaIIelingthe eventual extension of agriculture from Easter Island's coastal Iowlands to its uplands, and
from the Mimbres floodplain to the hills, COpan's inhabitants also expanded from the floodplain to the
more fragile hill slopes, leaving them with a larger population to feed when the agricultural boom in the
hills went bust. Like Easter Island chiefs erecting ever larger statues, eventually crowned by pukao, and
Iike Anasazi elite treatingthemselves to necklaces of 2,000 turquoise beads, Maya kings sought to outdo
each other with more and more impressive tempies, covered with thicker and thicker piaster-
reminiscent in turn of the extravagant conspicuous
w
w
w
.exam
race.com
consumption by modern American CEOs. The passivity of Easter chiefs and Maya kings in the face of the
real big threats to their societies completes our list of disquieting parallels.
86. Accordingto the passage, which of the following best represents the factor that has been cited by
the author in the context of Rwanda and Haiti?
(1) Various ethnic groups competingfor land and other resources
(2) Various ethnic groups competingfor limited land resources
(3) Various ethnic groups fighting wit each other
(4) Various ethnic groups competingfor political power
(5) Various ethnic groups fighting for their identity
87. By an anthropogenic drought, the author means CJ(1) A drought caused by lack of rains.
(2) A drought caused due to deforestation ~.., +
(3) A drought caused by failure to prevent brackenferns from oveu~lields.
(4) A drought caused by actions of human beings.
(5) A drought caused by elimate changes. ~
88. Accordingto the passage, the drought at thetime ~;u:;sehad a different impact compared
to the droughts earlier because ~
(1)The Maya kings continue to be extravagant§ com on people were suffering.
(2) It happened at thetime of collapse ~ol a <hip • ong Mayas.
(3) It happened when the Maya popula n upied all available land suited for agriculture.
(4) It was foilowed by internecina :w ar m g Mayans.
(5) Irreversible environmental d · d to this drought.
89. Accordingto the autho.a#t ifficult to explain the reasons for Maya collapse?
(1) COpan inhabitan de~~records of that period.
(2) tat• n and hillside erosion have wiped out all traces of the Maya kingdom.
(3) Mayas do not provide any consistent evidence.
(4) sibleto ascertain which ofthe factors best explains as to why theMaya
llapsed.
1"':~~ factor has not been cited as one of the factors causingthe collapse of Maya society?
vironmental degradation dueto excess population
5ocial collapse dueto excess population
Increased warfare among Maya people
dimate change
Obsession of Maya population with their own short-term concerns.

Contenu connexe

Tendances

4 ma0 3h_que_20150106
4 ma0 3h_que_20150106 4 ma0 3h_que_20150106
4 ma0 3h_que_20150106 AnneRostom
 
4 ma0 3hr_que_20140514
4 ma0 3hr_que_20140514 4 ma0 3hr_que_20140514
4 ma0 3hr_que_20140514 AnneRostom
 
Chapter 5 HW Answers
Chapter 5 HW AnswersChapter 5 HW Answers
Chapter 5 HW AnswersJenny Hubbard
 
Upcat math 2014 original
Upcat math 2014 originalUpcat math 2014 original
Upcat math 2014 originalMark Garrido
 
Upcat math 2014 solution
Upcat math 2014 solutionUpcat math 2014 solution
Upcat math 2014 solutionMark Garrido
 
Math vs. zombies (A Quiz Bowl on Polynomials)
Math vs. zombies (A Quiz Bowl on Polynomials)Math vs. zombies (A Quiz Bowl on Polynomials)
Math vs. zombies (A Quiz Bowl on Polynomials)jeshajomarie
 
CCA Chapter 4 HW Answers
CCA Chapter 4 HW AnswersCCA Chapter 4 HW Answers
CCA Chapter 4 HW AnswersJenny Hubbard
 
Chapter 6 HW Answers
Chapter 6 HW AnswersChapter 6 HW Answers
Chapter 6 HW AnswersJenny Hubbard
 
General education-mathematics-3
General education-mathematics-3General education-mathematics-3
General education-mathematics-3John Paul Intano
 
Chapter 2 Linear Functions
Chapter 2 Linear FunctionsChapter 2 Linear Functions
Chapter 2 Linear FunctionsJenny Hubbard
 
Maths Help Powerpoint / Revision Help
Maths Help Powerpoint / Revision Help Maths Help Powerpoint / Revision Help
Maths Help Powerpoint / Revision Help livylou0103
 

Tendances (20)

4 ma0 3h_que_20150106
4 ma0 3h_que_20150106 4 ma0 3h_que_20150106
4 ma0 3h_que_20150106
 
Maths quiz
Maths quizMaths quiz
Maths quiz
 
07 practice paper_4_h_set_a
07 practice paper_4_h_set_a07 practice paper_4_h_set_a
07 practice paper_4_h_set_a
 
4 ma0 3hr_que_20140514
4 ma0 3hr_que_20140514 4 ma0 3hr_que_20140514
4 ma0 3hr_que_20140514
 
Maths Quiz
Maths QuizMaths Quiz
Maths Quiz
 
Chapter 5 HW Answers
Chapter 5 HW AnswersChapter 5 HW Answers
Chapter 5 HW Answers
 
Maths quiz 6 8
Maths quiz 6 8Maths quiz 6 8
Maths quiz 6 8
 
Upcat math 2014 original
Upcat math 2014 originalUpcat math 2014 original
Upcat math 2014 original
 
Upcat math 2014
Upcat math 2014Upcat math 2014
Upcat math 2014
 
Upcat math 2014 solution
Upcat math 2014 solutionUpcat math 2014 solution
Upcat math 2014 solution
 
Math vs. zombies (A Quiz Bowl on Polynomials)
Math vs. zombies (A Quiz Bowl on Polynomials)Math vs. zombies (A Quiz Bowl on Polynomials)
Math vs. zombies (A Quiz Bowl on Polynomials)
 
CCA Chapter 7
CCA Chapter 7CCA Chapter 7
CCA Chapter 7
 
CCA Chapter 4 HW Answers
CCA Chapter 4 HW AnswersCCA Chapter 4 HW Answers
CCA Chapter 4 HW Answers
 
Chapter 6 HW Answers
Chapter 6 HW AnswersChapter 6 HW Answers
Chapter 6 HW Answers
 
General education-mathematics-3
General education-mathematics-3General education-mathematics-3
General education-mathematics-3
 
Chapter 2 Linear Functions
Chapter 2 Linear FunctionsChapter 2 Linear Functions
Chapter 2 Linear Functions
 
1.1 Math Quiz
1.1 Math Quiz1.1 Math Quiz
1.1 Math Quiz
 
Mega pp -_v1
Mega pp -_v1Mega pp -_v1
Mega pp -_v1
 
Maths Help Powerpoint / Revision Help
Maths Help Powerpoint / Revision Help Maths Help Powerpoint / Revision Help
Maths Help Powerpoint / Revision Help
 
Chapter 1 Functions
Chapter 1 FunctionsChapter 1 Functions
Chapter 1 Functions
 

En vedette

データベース技術 4(Database_4)
データベース技術 4(Database_4)データベース技術 4(Database_4)
データベース技術 4(Database_4)Yuka Obu
 
Interview to Ballet AL ARAB - Mendoza - Argentina
Interview to Ballet AL ARAB - Mendoza - ArgentinaInterview to Ballet AL ARAB - Mendoza - Argentina
Interview to Ballet AL ARAB - Mendoza - ArgentinaSaidHamdan
 
Solar & hydroelectric energy
Solar & hydroelectric energySolar & hydroelectric energy
Solar & hydroelectric energy2tgd
 
Anti platelets- facts and controversies
Anti platelets- facts and controversies Anti platelets- facts and controversies
Anti platelets- facts and controversies Mohammad Tanvir Islam
 
Everyone is a leader
Everyone is a leaderEveryone is a leader
Everyone is a leadercullmartin
 
Codes and conventions of music mag covers
Codes and conventions of music mag coversCodes and conventions of music mag covers
Codes and conventions of music mag coversShobyone
 
Some Pry Features
Some Pry FeaturesSome Pry Features
Some Pry FeaturesYann VERY
 
ULMAN GUI Specifications
ULMAN GUI SpecificationsULMAN GUI Specifications
ULMAN GUI Specificationsadil raja
 
Accounting services
Accounting servicesAccounting services
Accounting serviceschris75308
 
Introducing networks
Introducing networksIntroducing networks
Introducing networksMansoor Shar
 

En vedette (20)

กสพท. ภาษาอังกฤษ 2556
กสพท. ภาษาอังกฤษ 2556กสพท. ภาษาอังกฤษ 2556
กสพท. ภาษาอังกฤษ 2556
 
Avon opportunity
Avon opportunity Avon opportunity
Avon opportunity
 
IXP SDK
IXP SDKIXP SDK
IXP SDK
 
データベース技術 4(Database_4)
データベース技術 4(Database_4)データベース技術 4(Database_4)
データベース技術 4(Database_4)
 
Interview to Ballet AL ARAB - Mendoza - Argentina
Interview to Ballet AL ARAB - Mendoza - ArgentinaInterview to Ballet AL ARAB - Mendoza - Argentina
Interview to Ballet AL ARAB - Mendoza - Argentina
 
Solar & hydroelectric energy
Solar & hydroelectric energySolar & hydroelectric energy
Solar & hydroelectric energy
 
Anti platelets- facts and controversies
Anti platelets- facts and controversies Anti platelets- facts and controversies
Anti platelets- facts and controversies
 
Everyone is a leader
Everyone is a leaderEveryone is a leader
Everyone is a leader
 
A battery charger is a device that can restore the charger to a battery
A battery charger is a device that can restore the charger to a batteryA battery charger is a device that can restore the charger to a battery
A battery charger is a device that can restore the charger to a battery
 
IMPORT FROM TURKEY
IMPORT FROM TURKEYIMPORT FROM TURKEY
IMPORT FROM TURKEY
 
current resume
current resumecurrent resume
current resume
 
Codes and conventions of music mag covers
Codes and conventions of music mag coversCodes and conventions of music mag covers
Codes and conventions of music mag covers
 
Compt day
Compt dayCompt day
Compt day
 
Some Pry Features
Some Pry FeaturesSome Pry Features
Some Pry Features
 
Active pasive voice continuous
Active pasive voice continuousActive pasive voice continuous
Active pasive voice continuous
 
ULMAN GUI Specifications
ULMAN GUI SpecificationsULMAN GUI Specifications
ULMAN GUI Specifications
 
B erfikir kreatif dan inovasi pelayanan publik kota tarakan
B erfikir kreatif dan inovasi pelayanan publik kota tarakanB erfikir kreatif dan inovasi pelayanan publik kota tarakan
B erfikir kreatif dan inovasi pelayanan publik kota tarakan
 
вода
водавода
вода
 
Accounting services
Accounting servicesAccounting services
Accounting services
 
Introducing networks
Introducing networksIntroducing networks
Introducing networks
 

Similaire à Cat Paper 2008 by MBA Institutes in Mumbai

Cat 2008 paper with key proton training solutions
Cat 2008   paper with key proton training solutionsCat 2008   paper with key proton training solutions
Cat 2008 paper with key proton training solutionsProton Training Solutions
 
CAT 2005 Previous Year Question Paper with Answer Key
CAT 2005 Previous Year Question Paper with Answer KeyCAT 2005 Previous Year Question Paper with Answer Key
CAT 2005 Previous Year Question Paper with Answer KeyEneutron
 
Cat 2005 paper with key proton training solutions
Cat 2005   paper with key proton training solutionsCat 2005   paper with key proton training solutions
Cat 2005 paper with key proton training solutionsProton Training Solutions
 
Class 10 Cbse Maths 2010 Sample Paper Model 2
Class 10 Cbse Maths 2010 Sample Paper Model 2Class 10 Cbse Maths 2010 Sample Paper Model 2
Class 10 Cbse Maths 2010 Sample Paper Model 2Sunaina Rawat
 
Class 10 Cbse Maths Question Paper Term 1 2011
Class 10 Cbse Maths Question Paper Term 1 2011Class 10 Cbse Maths Question Paper Term 1 2011
Class 10 Cbse Maths Question Paper Term 1 2011Sunaina Rawat
 
357402584 242463893-pt3-trial-mathematics-pdf
357402584 242463893-pt3-trial-mathematics-pdf357402584 242463893-pt3-trial-mathematics-pdf
357402584 242463893-pt3-trial-mathematics-pdfSyahidaZamri1
 
Cat 2007 paper with key proton training solutions
Cat 2007   paper with key proton training solutions Cat 2007   paper with key proton training solutions
Cat 2007 paper with key proton training solutions Proton Training Solutions
 
Class 10 Cbse Maths 2010 Sample Paper Model 1
Class 10 Cbse Maths 2010 Sample Paper Model 1Class 10 Cbse Maths 2010 Sample Paper Model 1
Class 10 Cbse Maths 2010 Sample Paper Model 1Sunaina Rawat
 
Cbse sample-papers-class-10-maths-sa-ii-solved-2
Cbse sample-papers-class-10-maths-sa-ii-solved-2Cbse sample-papers-class-10-maths-sa-ii-solved-2
Cbse sample-papers-class-10-maths-sa-ii-solved-2gyanpub
 
4 ma0 3h_que_20140514
4 ma0 3h_que_20140514 4 ma0 3h_que_20140514
4 ma0 3h_que_20140514 AnneRostom
 
HIGHSCHOOL MATH REVIEWER
HIGHSCHOOL MATH REVIEWERHIGHSCHOOL MATH REVIEWER
HIGHSCHOOL MATH REVIEWERJohn Labrador
 
Higher nov 2008_p1old
Higher nov 2008_p1oldHigher nov 2008_p1old
Higher nov 2008_p1oldybamary
 
Class 9 Cbse Maths Sample Paper Term 1 Model 1
Class 9 Cbse Maths Sample Paper Term 1 Model 1Class 9 Cbse Maths Sample Paper Term 1 Model 1
Class 9 Cbse Maths Sample Paper Term 1 Model 1Sunaina Rawat
 
MATHEMATICS MODEL QUESTION PAPER FOR CBSE CLASS 10
MATHEMATICS MODEL QUESTION PAPER FOR CBSE CLASS 10MATHEMATICS MODEL QUESTION PAPER FOR CBSE CLASS 10
MATHEMATICS MODEL QUESTION PAPER FOR CBSE CLASS 10Bagalkot
 

Similaire à Cat Paper 2008 by MBA Institutes in Mumbai (20)

Cat 2008 paper with key proton training solutions
Cat 2008   paper with key proton training solutionsCat 2008   paper with key proton training solutions
Cat 2008 paper with key proton training solutions
 
CAT 2005 Previous Year Question Paper with Answer Key
CAT 2005 Previous Year Question Paper with Answer KeyCAT 2005 Previous Year Question Paper with Answer Key
CAT 2005 Previous Year Question Paper with Answer Key
 
CAT- 2008
CAT- 2008 CAT- 2008
CAT- 2008
 
Cat 2005 paper with key proton training solutions
Cat 2005   paper with key proton training solutionsCat 2005   paper with key proton training solutions
Cat 2005 paper with key proton training solutions
 
Class 10 Cbse Maths 2010 Sample Paper Model 2
Class 10 Cbse Maths 2010 Sample Paper Model 2Class 10 Cbse Maths 2010 Sample Paper Model 2
Class 10 Cbse Maths 2010 Sample Paper Model 2
 
Class 10 Cbse Maths Question Paper Term 1 2011
Class 10 Cbse Maths Question Paper Term 1 2011Class 10 Cbse Maths Question Paper Term 1 2011
Class 10 Cbse Maths Question Paper Term 1 2011
 
357402584 242463893-pt3-trial-mathematics-pdf
357402584 242463893-pt3-trial-mathematics-pdf357402584 242463893-pt3-trial-mathematics-pdf
357402584 242463893-pt3-trial-mathematics-pdf
 
Cat 2007 paper with key proton training solutions
Cat 2007   paper with key proton training solutions Cat 2007   paper with key proton training solutions
Cat 2007 paper with key proton training solutions
 
Class 10 Cbse Maths 2010 Sample Paper Model 1
Class 10 Cbse Maths 2010 Sample Paper Model 1Class 10 Cbse Maths 2010 Sample Paper Model 1
Class 10 Cbse Maths 2010 Sample Paper Model 1
 
X maths 1
X maths 1X maths 1
X maths 1
 
Pts m-10
Pts m-10Pts m-10
Pts m-10
 
Cbse sample-papers-class-10-maths-sa-ii-solved-2
Cbse sample-papers-class-10-maths-sa-ii-solved-2Cbse sample-papers-class-10-maths-sa-ii-solved-2
Cbse sample-papers-class-10-maths-sa-ii-solved-2
 
4 ma0 3h_que_20140514
4 ma0 3h_que_20140514 4 ma0 3h_que_20140514
4 ma0 3h_que_20140514
 
HIGHSCHOOL MATH REVIEWER
HIGHSCHOOL MATH REVIEWERHIGHSCHOOL MATH REVIEWER
HIGHSCHOOL MATH REVIEWER
 
Higher nov 2008_p1old
Higher nov 2008_p1oldHigher nov 2008_p1old
Higher nov 2008_p1old
 
Cat sample-1
Cat sample-1Cat sample-1
Cat sample-1
 
no maths
no mathsno maths
no maths
 
Class 9 Cbse Maths Sample Paper Term 1 Model 1
Class 9 Cbse Maths Sample Paper Term 1 Model 1Class 9 Cbse Maths Sample Paper Term 1 Model 1
Class 9 Cbse Maths Sample Paper Term 1 Model 1
 
MATHEMATICS MODEL QUESTION PAPER FOR CBSE CLASS 10
MATHEMATICS MODEL QUESTION PAPER FOR CBSE CLASS 10MATHEMATICS MODEL QUESTION PAPER FOR CBSE CLASS 10
MATHEMATICS MODEL QUESTION PAPER FOR CBSE CLASS 10
 
MFCS UNIT-III.pptx
MFCS UNIT-III.pptxMFCS UNIT-III.pptx
MFCS UNIT-III.pptx
 

Plus de seomiamia

MBA classes in Mumbai
MBA classes in MumbaiMBA classes in Mumbai
MBA classes in Mumbaiseomiamia
 
Important Mock test TISS- MBA
Important Mock test TISS- MBA Important Mock test TISS- MBA
Important Mock test TISS- MBA seomiamia
 
Important GK Compendium for MBA Exams
Important GK Compendium for MBA ExamsImportant GK Compendium for MBA Exams
Important GK Compendium for MBA Examsseomiamia
 
Important Class X Science Paper
Important Class X Science PaperImportant Class X Science Paper
Important Class X Science Paperseomiamia
 
Important Accounts Paper - 2015 - Commerce
Important Accounts Paper - 2015 - CommerceImportant Accounts Paper - 2015 - Commerce
Important Accounts Paper - 2015 - Commerceseomiamia
 
Important GK on Modern India History for MBA
Important GK on Modern India History for MBA Important GK on Modern India History for MBA
Important GK on Modern India History for MBA seomiamia
 
Important paper for Commerce and Accountancy
Important paper for Commerce and Accountancy Important paper for Commerce and Accountancy
Important paper for Commerce and Accountancy seomiamia
 
Taxation Paper 4 - IPCC May 2012
Taxation Paper 4  - IPCC May 2012Taxation Paper 4  - IPCC May 2012
Taxation Paper 4 - IPCC May 2012seomiamia
 
CMAT Sample Paper 1 - MBA Classes
CMAT Sample Paper 1 - MBA ClassesCMAT Sample Paper 1 - MBA Classes
CMAT Sample Paper 1 - MBA Classesseomiamia
 
Gk compendium for non cat exams 2015 Part 2
Gk compendium for non cat exams  2015 Part 2Gk compendium for non cat exams  2015 Part 2
Gk compendium for non cat exams 2015 Part 2seomiamia
 
Gk Compendium for non Cat exams 2015
Gk Compendium for non Cat exams  2015Gk Compendium for non Cat exams  2015
Gk Compendium for non Cat exams 2015seomiamia
 
CMAT GK Question Bank 2014
CMAT GK Question Bank 2014CMAT GK Question Bank 2014
CMAT GK Question Bank 2014seomiamia
 
Down town restaurants map
Down town restaurants mapDown town restaurants map
Down town restaurants mapseomiamia
 
Restaurant trends 2013
Restaurant trends 2013Restaurant trends 2013
Restaurant trends 2013seomiamia
 
IIFT Addendum 2015 for MBA Classes
IIFT Addendum 2015 for MBA ClassesIIFT Addendum 2015 for MBA Classes
IIFT Addendum 2015 for MBA Classesseomiamia
 
IPCC Question Paper NOvember 2013
IPCC Question Paper NOvember 2013IPCC Question Paper NOvember 2013
IPCC Question Paper NOvember 2013seomiamia
 
IIFT 2014 Solutions
IIFT 2014 SolutionsIIFT 2014 Solutions
IIFT 2014 Solutionsseomiamia
 
IIFT Addendum 2015 for MBA
IIFT Addendum  2015 for MBAIIFT Addendum  2015 for MBA
IIFT Addendum 2015 for MBAseomiamia
 
IIFT 2014 question paper for MBA
IIFT 2014 question paper for MBAIIFT 2014 question paper for MBA
IIFT 2014 question paper for MBAseomiamia
 
Gk Compendium for MBA Exams 2015
Gk Compendium for MBA Exams 2015Gk Compendium for MBA Exams 2015
Gk Compendium for MBA Exams 2015seomiamia
 

Plus de seomiamia (20)

MBA classes in Mumbai
MBA classes in MumbaiMBA classes in Mumbai
MBA classes in Mumbai
 
Important Mock test TISS- MBA
Important Mock test TISS- MBA Important Mock test TISS- MBA
Important Mock test TISS- MBA
 
Important GK Compendium for MBA Exams
Important GK Compendium for MBA ExamsImportant GK Compendium for MBA Exams
Important GK Compendium for MBA Exams
 
Important Class X Science Paper
Important Class X Science PaperImportant Class X Science Paper
Important Class X Science Paper
 
Important Accounts Paper - 2015 - Commerce
Important Accounts Paper - 2015 - CommerceImportant Accounts Paper - 2015 - Commerce
Important Accounts Paper - 2015 - Commerce
 
Important GK on Modern India History for MBA
Important GK on Modern India History for MBA Important GK on Modern India History for MBA
Important GK on Modern India History for MBA
 
Important paper for Commerce and Accountancy
Important paper for Commerce and Accountancy Important paper for Commerce and Accountancy
Important paper for Commerce and Accountancy
 
Taxation Paper 4 - IPCC May 2012
Taxation Paper 4  - IPCC May 2012Taxation Paper 4  - IPCC May 2012
Taxation Paper 4 - IPCC May 2012
 
CMAT Sample Paper 1 - MBA Classes
CMAT Sample Paper 1 - MBA ClassesCMAT Sample Paper 1 - MBA Classes
CMAT Sample Paper 1 - MBA Classes
 
Gk compendium for non cat exams 2015 Part 2
Gk compendium for non cat exams  2015 Part 2Gk compendium for non cat exams  2015 Part 2
Gk compendium for non cat exams 2015 Part 2
 
Gk Compendium for non Cat exams 2015
Gk Compendium for non Cat exams  2015Gk Compendium for non Cat exams  2015
Gk Compendium for non Cat exams 2015
 
CMAT GK Question Bank 2014
CMAT GK Question Bank 2014CMAT GK Question Bank 2014
CMAT GK Question Bank 2014
 
Down town restaurants map
Down town restaurants mapDown town restaurants map
Down town restaurants map
 
Restaurant trends 2013
Restaurant trends 2013Restaurant trends 2013
Restaurant trends 2013
 
IIFT Addendum 2015 for MBA Classes
IIFT Addendum 2015 for MBA ClassesIIFT Addendum 2015 for MBA Classes
IIFT Addendum 2015 for MBA Classes
 
IPCC Question Paper NOvember 2013
IPCC Question Paper NOvember 2013IPCC Question Paper NOvember 2013
IPCC Question Paper NOvember 2013
 
IIFT 2014 Solutions
IIFT 2014 SolutionsIIFT 2014 Solutions
IIFT 2014 Solutions
 
IIFT Addendum 2015 for MBA
IIFT Addendum  2015 for MBAIIFT Addendum  2015 for MBA
IIFT Addendum 2015 for MBA
 
IIFT 2014 question paper for MBA
IIFT 2014 question paper for MBAIIFT 2014 question paper for MBA
IIFT 2014 question paper for MBA
 
Gk Compendium for MBA Exams 2015
Gk Compendium for MBA Exams 2015Gk Compendium for MBA Exams 2015
Gk Compendium for MBA Exams 2015
 

Dernier

A Critique of the Proposed National Education Policy Reform
A Critique of the Proposed National Education Policy ReformA Critique of the Proposed National Education Policy Reform
A Critique of the Proposed National Education Policy ReformChameera Dedduwage
 
SOCIAL AND HISTORICAL CONTEXT - LFTVD.pptx
SOCIAL AND HISTORICAL CONTEXT - LFTVD.pptxSOCIAL AND HISTORICAL CONTEXT - LFTVD.pptx
SOCIAL AND HISTORICAL CONTEXT - LFTVD.pptxiammrhaywood
 
APM Welcome, APM North West Network Conference, Synergies Across Sectors
APM Welcome, APM North West Network Conference, Synergies Across SectorsAPM Welcome, APM North West Network Conference, Synergies Across Sectors
APM Welcome, APM North West Network Conference, Synergies Across SectorsAssociation for Project Management
 
Beyond the EU: DORA and NIS 2 Directive's Global Impact
Beyond the EU: DORA and NIS 2 Directive's Global ImpactBeyond the EU: DORA and NIS 2 Directive's Global Impact
Beyond the EU: DORA and NIS 2 Directive's Global ImpactPECB
 
CARE OF CHILD IN INCUBATOR..........pptx
CARE OF CHILD IN INCUBATOR..........pptxCARE OF CHILD IN INCUBATOR..........pptx
CARE OF CHILD IN INCUBATOR..........pptxGaneshChakor2
 
Accessible design: Minimum effort, maximum impact
Accessible design: Minimum effort, maximum impactAccessible design: Minimum effort, maximum impact
Accessible design: Minimum effort, maximum impactdawncurless
 
Arihant handbook biology for class 11 .pdf
Arihant handbook biology for class 11 .pdfArihant handbook biology for class 11 .pdf
Arihant handbook biology for class 11 .pdfchloefrazer622
 
Sports & Fitness Value Added Course FY..
Sports & Fitness Value Added Course FY..Sports & Fitness Value Added Course FY..
Sports & Fitness Value Added Course FY..Disha Kariya
 
The basics of sentences session 2pptx copy.pptx
The basics of sentences session 2pptx copy.pptxThe basics of sentences session 2pptx copy.pptx
The basics of sentences session 2pptx copy.pptxheathfieldcps1
 
social pharmacy d-pharm 1st year by Pragati K. Mahajan
social pharmacy d-pharm 1st year by Pragati K. Mahajansocial pharmacy d-pharm 1st year by Pragati K. Mahajan
social pharmacy d-pharm 1st year by Pragati K. Mahajanpragatimahajan3
 
Russian Call Girls in Andheri Airport Mumbai WhatsApp 9167673311 💞 Full Nigh...
Russian Call Girls in Andheri Airport Mumbai WhatsApp  9167673311 💞 Full Nigh...Russian Call Girls in Andheri Airport Mumbai WhatsApp  9167673311 💞 Full Nigh...
Russian Call Girls in Andheri Airport Mumbai WhatsApp 9167673311 💞 Full Nigh...Pooja Nehwal
 
Advanced Views - Calendar View in Odoo 17
Advanced Views - Calendar View in Odoo 17Advanced Views - Calendar View in Odoo 17
Advanced Views - Calendar View in Odoo 17Celine George
 
Sanyam Choudhary Chemistry practical.pdf
Sanyam Choudhary Chemistry practical.pdfSanyam Choudhary Chemistry practical.pdf
Sanyam Choudhary Chemistry practical.pdfsanyamsingh5019
 
Grant Readiness 101 TechSoup and Remy Consulting
Grant Readiness 101 TechSoup and Remy ConsultingGrant Readiness 101 TechSoup and Remy Consulting
Grant Readiness 101 TechSoup and Remy ConsultingTechSoup
 
Ecosystem Interactions Class Discussion Presentation in Blue Green Lined Styl...
Ecosystem Interactions Class Discussion Presentation in Blue Green Lined Styl...Ecosystem Interactions Class Discussion Presentation in Blue Green Lined Styl...
Ecosystem Interactions Class Discussion Presentation in Blue Green Lined Styl...fonyou31
 
JAPAN: ORGANISATION OF PMDA, PHARMACEUTICAL LAWS & REGULATIONS, TYPES OF REGI...
JAPAN: ORGANISATION OF PMDA, PHARMACEUTICAL LAWS & REGULATIONS, TYPES OF REGI...JAPAN: ORGANISATION OF PMDA, PHARMACEUTICAL LAWS & REGULATIONS, TYPES OF REGI...
JAPAN: ORGANISATION OF PMDA, PHARMACEUTICAL LAWS & REGULATIONS, TYPES OF REGI...anjaliyadav012327
 
The byproduct of sericulture in different industries.pptx
The byproduct of sericulture in different industries.pptxThe byproduct of sericulture in different industries.pptx
The byproduct of sericulture in different industries.pptxShobhayan Kirtania
 
BASLIQ CURRENT LOOKBOOK LOOKBOOK(1) (1).pdf
BASLIQ CURRENT LOOKBOOK  LOOKBOOK(1) (1).pdfBASLIQ CURRENT LOOKBOOK  LOOKBOOK(1) (1).pdf
BASLIQ CURRENT LOOKBOOK LOOKBOOK(1) (1).pdfSoniaTolstoy
 
Nutritional Needs Presentation - HLTH 104
Nutritional Needs Presentation - HLTH 104Nutritional Needs Presentation - HLTH 104
Nutritional Needs Presentation - HLTH 104misteraugie
 

Dernier (20)

A Critique of the Proposed National Education Policy Reform
A Critique of the Proposed National Education Policy ReformA Critique of the Proposed National Education Policy Reform
A Critique of the Proposed National Education Policy Reform
 
SOCIAL AND HISTORICAL CONTEXT - LFTVD.pptx
SOCIAL AND HISTORICAL CONTEXT - LFTVD.pptxSOCIAL AND HISTORICAL CONTEXT - LFTVD.pptx
SOCIAL AND HISTORICAL CONTEXT - LFTVD.pptx
 
APM Welcome, APM North West Network Conference, Synergies Across Sectors
APM Welcome, APM North West Network Conference, Synergies Across SectorsAPM Welcome, APM North West Network Conference, Synergies Across Sectors
APM Welcome, APM North West Network Conference, Synergies Across Sectors
 
Beyond the EU: DORA and NIS 2 Directive's Global Impact
Beyond the EU: DORA and NIS 2 Directive's Global ImpactBeyond the EU: DORA and NIS 2 Directive's Global Impact
Beyond the EU: DORA and NIS 2 Directive's Global Impact
 
CARE OF CHILD IN INCUBATOR..........pptx
CARE OF CHILD IN INCUBATOR..........pptxCARE OF CHILD IN INCUBATOR..........pptx
CARE OF CHILD IN INCUBATOR..........pptx
 
Accessible design: Minimum effort, maximum impact
Accessible design: Minimum effort, maximum impactAccessible design: Minimum effort, maximum impact
Accessible design: Minimum effort, maximum impact
 
Arihant handbook biology for class 11 .pdf
Arihant handbook biology for class 11 .pdfArihant handbook biology for class 11 .pdf
Arihant handbook biology for class 11 .pdf
 
Sports & Fitness Value Added Course FY..
Sports & Fitness Value Added Course FY..Sports & Fitness Value Added Course FY..
Sports & Fitness Value Added Course FY..
 
The basics of sentences session 2pptx copy.pptx
The basics of sentences session 2pptx copy.pptxThe basics of sentences session 2pptx copy.pptx
The basics of sentences session 2pptx copy.pptx
 
social pharmacy d-pharm 1st year by Pragati K. Mahajan
social pharmacy d-pharm 1st year by Pragati K. Mahajansocial pharmacy d-pharm 1st year by Pragati K. Mahajan
social pharmacy d-pharm 1st year by Pragati K. Mahajan
 
Mattingly "AI & Prompt Design: The Basics of Prompt Design"
Mattingly "AI & Prompt Design: The Basics of Prompt Design"Mattingly "AI & Prompt Design: The Basics of Prompt Design"
Mattingly "AI & Prompt Design: The Basics of Prompt Design"
 
Russian Call Girls in Andheri Airport Mumbai WhatsApp 9167673311 💞 Full Nigh...
Russian Call Girls in Andheri Airport Mumbai WhatsApp  9167673311 💞 Full Nigh...Russian Call Girls in Andheri Airport Mumbai WhatsApp  9167673311 💞 Full Nigh...
Russian Call Girls in Andheri Airport Mumbai WhatsApp 9167673311 💞 Full Nigh...
 
Advanced Views - Calendar View in Odoo 17
Advanced Views - Calendar View in Odoo 17Advanced Views - Calendar View in Odoo 17
Advanced Views - Calendar View in Odoo 17
 
Sanyam Choudhary Chemistry practical.pdf
Sanyam Choudhary Chemistry practical.pdfSanyam Choudhary Chemistry practical.pdf
Sanyam Choudhary Chemistry practical.pdf
 
Grant Readiness 101 TechSoup and Remy Consulting
Grant Readiness 101 TechSoup and Remy ConsultingGrant Readiness 101 TechSoup and Remy Consulting
Grant Readiness 101 TechSoup and Remy Consulting
 
Ecosystem Interactions Class Discussion Presentation in Blue Green Lined Styl...
Ecosystem Interactions Class Discussion Presentation in Blue Green Lined Styl...Ecosystem Interactions Class Discussion Presentation in Blue Green Lined Styl...
Ecosystem Interactions Class Discussion Presentation in Blue Green Lined Styl...
 
JAPAN: ORGANISATION OF PMDA, PHARMACEUTICAL LAWS & REGULATIONS, TYPES OF REGI...
JAPAN: ORGANISATION OF PMDA, PHARMACEUTICAL LAWS & REGULATIONS, TYPES OF REGI...JAPAN: ORGANISATION OF PMDA, PHARMACEUTICAL LAWS & REGULATIONS, TYPES OF REGI...
JAPAN: ORGANISATION OF PMDA, PHARMACEUTICAL LAWS & REGULATIONS, TYPES OF REGI...
 
The byproduct of sericulture in different industries.pptx
The byproduct of sericulture in different industries.pptxThe byproduct of sericulture in different industries.pptx
The byproduct of sericulture in different industries.pptx
 
BASLIQ CURRENT LOOKBOOK LOOKBOOK(1) (1).pdf
BASLIQ CURRENT LOOKBOOK  LOOKBOOK(1) (1).pdfBASLIQ CURRENT LOOKBOOK  LOOKBOOK(1) (1).pdf
BASLIQ CURRENT LOOKBOOK LOOKBOOK(1) (1).pdf
 
Nutritional Needs Presentation - HLTH 104
Nutritional Needs Presentation - HLTH 104Nutritional Needs Presentation - HLTH 104
Nutritional Needs Presentation - HLTH 104
 

Cat Paper 2008 by MBA Institutes in Mumbai

  • 1. w w w .exam race.com INSTRUCTIONS Beforethetest: 1. DO NOT REMOVE THE SEALS OFTHE I>LASTIC ENVELOI>E OF THIS BOOKLET UNTILTHE SIGNALTO START IS GIVEN. 2. Keep onlythe Admit Card, pencil, eraser and sharpenerwith you. DO NOT KEEl> with you books, rulers, slide rulers, drawing instruments, calculators (including watch calculators), pagers, cellular phones, stop watches or any other device or loose paper. These should be left at a place~ind icated by the invigilator. 3. Use only HB pencil to fill in the Answer sheet. 4. Enter in your Answer Sheet: (a) in Box 3,t Test Form Number that appears at the b( Stt his p ge, (b) in Box4, the Test Booklet Serial Numberthat appears atthe top ofthis page. s. Ensure that your personal data have been entered correctly on Side- II of th6 sh . 6. Ensure that you have entered your 7-digit Test Registration Number in Box 2 fthe nswer sheet correctly. Atthe start oftheTest: 0 + 1. As soon as the signal to start is given, open the Test Booklet.r ~ 2. ThisTest Booklet contains 36 pages, includingthe blan~"es~ately after opening the Test Booklet, verify that all the pages are printed properly d · order. If there is a pnoblem with your Test Booklet, immediately inform the invigiIator. ~Wi1 e rovid ed with a replacement. How to answer: ~ 1. 2. 3. 4. nsofthe 1nv1g1lator. Cand1datesfound v1olatmg the 1nstruwonsWill be 2. You may retain this Test Booklet with you. Candidares giving assisrance orseeking/receivingfrom anysource in answering quesrions or copying in any manner in rhe resr willforfeir rheir chances of being consideredfor admission. Such candidares willforfeir rhe righ r ro rhe scorecard. The resring aurhority reserves rherighr ro exdude any quesrion or quesrionsfrom rhis Tesr Booklerforfinal examinarion.
  • 2. w w w .exam race.com Test Form Number: SECTION I 1. A shop stores x kg of rice. The first customer buys half this amount pius half a kg of rice. The second customer buys half the remaining amount plus half a kg of rice. Then the third customer also buys haIf the remaining amount plus half a kg of rice. Thereafter, no rice is Ieft in the shop. Which of the following best describes the value of x? (1) Hx~6 (2)5~x~8 (3)9~x~12 (4)11~x~14 (5) 1Hx~18 Directions for Questions 2 and 3: Let J(x) = a)+ bx + c, where a, b and c are certain constants and a " 0. It is known that 3 is a root of f(x) = 0. 2. What is the other root ofJ(x) = 0? (1) -7 (2) -4 (3) 2 (4) 6 3. What 1sthe value of a+ b+ c? ~ (1) 9 (2) 14 (3) 13 (4) 37 j"'Vbedeterm1ned IS (1) 78 (2) 19 (3) 20 f'l:J- 77 0~ ~· (5) 22
  • 3. w w w .exam race.com The figure below shows the pian of a town. The streets are at right angles to each other. A rectangular park (P) is situated inside the town with a diagonal road running through it. There is also a prohibited region (D) in the town. (' BCJ '· NoO•m "'~h~b'"''' from h~how• • '" h~'"'" <,L::,~:,.OnooIt• number of poss1ble shortest paths that she can choose 1s ~- "XJ (1) 60 (2) 75 (3) 45 (4) 90 ~U' 6. Neelam ndes her b1cycle from her house at -~atc; v1a B takmg the shortest path. Then the number of poss1ble shortest paths that sh~~~lub (I)Um (>) "" (>~()(.) """ 0) '" Is dievalueof - (q 0 ~ ~ "' i (<! 1 "' ........._ ~""I, >, .... " •~~""'"'blookbo.rn Tho folio~"''""""' "lh~~'"""'" "•" In each repetition, any two numbers, say a and b, currently on the blackboard are erased and a new number a+ b- 1 is written. What wiII be the number left on the board at the end? (1) 820 (2) 821 (3) 781 (4) 819 (5) 780
  • 4. w w w .exam race.com = seed(s(n)), otherwise, wheres(n) indicates the sum of digits of n. For example, seed(7) = 7, seed(248)= seed(2 + 4 + 8)= seed(14)= seed(1 + 4) = seed(5)= 5 etc. How many positive integers n, such that n < 500, wiII have seed(n) = 9? (1) 39 (2) 72 (3) 81 (4) 108 (5) 55 10. In a triangle ABc; the lengths of the sides AB and AC equal17.5 em and 9 em respectively. Le point on the line segment BC such that 1>0 is perpendicular to BC If 1>0 = 3 em, then what is th em) ofthe circle circumscribingthetriangle ABC? (1) 17.05 (2) 27.85 (3) 22.45 (4) 32.25 (5) 26.25 11. What arethe last two digits of 7"SJos? (1) 21 (2) 61 (3)01 (4)41 (5)81 •12. If the roots of the equation x'- a)+bx- c =0 are three consecutiv en what is the smallest possible value of b? many 5 F and H. Consider points P and Q, on L and inside ABCD, such that the angles APD and 20•. What is the ratio of the area ofABQCDP to the remaining area inside ABCD? (2)2+1/3 (3} 1fH</3 9 l (4)1+43 (5)243-1 17. Three consecutive positive integers are raised to the first, second and third powers respectively and then added. The sum so obtained is a perfect square whose square root equals the total of the three original integers. Which ofthe foil owing best describes the minimum, say m, of thesethree integers? (1) Hm~3 (2) 4~m~6 (3) Hm~9 (4) 10~m~12 (5) 13~m~15
  • 5. w w w .exam race.com 1 1 18.Find the sum 1+-+-+ 12 22 1 1 1+2+1 +.............+ 2 3 (1) :zoo&.~ 1 (2) 2007-- 2007 1 1 1+--+-- 20072 20012 19. Two circles, both of radii 1 em, intersect such that the circumference of each or~ ugh the centre of the circle ofthe other. What is the area (in sq em) of the intersecting regiv (1).!!-./3 (2) 2n+./3 (3)4n-./3 (4)411+ ./3 ,cftJ•34 32 32 32 .:v 20. Rahim plans to drive from city A to station c, at these~mper hour, to catch a train arriving there from B. He must reach Cat least 15 minutes b ""t arrival of thetrain. The train Ieaves B, located 500 km south of A at 8:00 am and travels at a ed km per hour. It is known that C is located between west and northwest of B, with BC at 6 o so, Cis located between south and southwest of A w1th AC at 30• to AB. The latest t1me by ~ must leave A and st1ll catch the tra1n 1s closest to (1)6:15 am (2) 6:30am (3) ..!,.] (4) 7:00am (5) 7:15am 21. Cons1der a nght c1rcular cA~us4 em and he1ght 10 em. A cyhnder 1s to be piaced 1ns1dethe oooo w<h ooo of''"i~'"fho "'"of fho oooo ''"' fho '"''" "'"'blo •~•I '""'~ area (in sq. em) of th cy der"+ 00 ~ "' (3) ~ (4) ":" (S) '':" ~
  • 6. w w w .exam race.com Five horses, Red, White, Grey, Black and Spotted participated in a race. 1>s per the rules of the race, the persons betting on the winning horse get four times the bet amaunt and those betting on the horse that came in second get thrice the bet amaunt. Moreover, the bet amaunt is returned to those betting on the horse that came in third, and the rest Iose the bet amount. Raju bets Rs. 3000, Rs. 2000 Rs. 1000 on Red, White and Black horses respectively and ends up with no profit and no loss. 22. Which ofthe following cannot be true? (1) At least two horses f1n1shed before Spotted (2) Red f1n1shed last (3) There were three horses between Black and Spotted (S) Grey came In second r ~ 23. Suppose, 1n add1t1on, 1t 1s known that Grey came 1n fourth. Then wh1ch of the f~ cannot be true? ~.. + (1) Spotted came in first CJ"'(2) Red finished last ((3 4 )) White came in second ~flj Black came in second (S) There was one horse between Black and White 0~ • ~
  • 7. w w w .exam race.com Marks (1) if Q can be answered from A alone but not from B alone. Marks (2) if Q can be answered from B alone but not from A alone. Marks (3) if Q can be answered from A alone as well as from B alone. Marks (4) if Q can be answered from A and B together but not from any of them alone. Marks (5) if Q cannot be answered even from A and B together. In a single elimination tournament, any player is eliminated with a single loss. The tourname multiple rounds subject to the foilowing rules: (a) If the number of players, say n, in any round is even, then the players aree ro Q n/2 pairs. The piayers in each pair play a match against each other and the winner mov on t ext round. (b) If the number of piayers, say n, in any round is odd, then one of them is · bye, that is, he automatically maves on to the next round. The remaining (n - 1) p3 are.f'ouped into (n - 1)/2 pairs. The piayers in each pair play a match against each other and n rs moves on to the next round. No piayer gets morethan one bye inthe entire tourna'1~ : Thus, if n is even, then n/2 players mave on to the next roun~~~d, then (n+1)/2 piayers mave on to the next round. The process is continued till the fittr~~ , 1ch obviously is played between two players. The winner in thefinal round is the champion f~u ment. 24. Q: What is the number of matches played '
  • 8. w w w .exam race.com SECTION II This section contains 25 questions Directions for Questions 26 to 28: Answer thefollowing questions based on the information given below: For admission to various affiliated colleges, a university conducts a written test with four different sections, each with a maximum of 50 marks. The following table gives the aggregate as welle.<::tth sectional cut-off marks fixed by six different colleges affiliated to the university. A stud..._......., College I College 3 College 4 College 6 S(~CtiOil ; 42 43 45 Se:tional Cut-off Marks St~ction H St·ction C 42 42 45 176 175 171 45 180 44 176 26. Ad1tya d1d not get a call ~eve a smgle college. What could be the max1mum aggregate marks obtained by h1m? 27. Bhama got c fr o! ges. What could be the minimum aggregate marks obtained by her? (1)181 ~2)1 3)184 (4)196 (5)190 (1)180 )18 (3)196 (4)176 (5)184 (2) 21 (3) 25 (4) 35 (5) 41
  • 9. w w w .exam race.com The bar chart below shows the revenue received, in million US Dollars (USD), from subscribers to a particular Internet service. The data covers the period 2003 to 2007 for the United States (US) and Europe. The bar chart also shows the estimate revenues from the subscription to this service for the period 2008 to 2010. .a .~ " 1000 900 800 700 600 500 400 300 200 'i 100 ;; "' 0 JJ 03 04 l ........... t~....... [ 1 i.l[m ~..·.l····l·k·~·;,rl:.•.•.•.·..·..•.·. 11.·········1~ us CJ· ········· !! I[ ~~[ ~.Europe tli ~,! i (! t~.... ·r IIi Ii1 ! .il .J![. J!l , •• OS 06 07 08 09 10 CJ"'Vear 29. While the subscnpt1on 1n Europe has been grow1ng stl!d~sthat of the US, the growth rate 1n Europe seems to be dechmng. Wh1ch of the follow ~~st to the percent change 1n growth rate of 2007 (over 2006) relative to the growth rate of 2 ver 04)? (1) 17 (2) 20 (3) 35 (4) 60 (.,.,_..... 30. The difference between the estf f :·m ed ~tion in Europe in 2008 and what it would have been if it were computed usingthe percenta ' rate of 2007 (over 2006), is closest to : (1) 50 (2) 80 (3) 20 0 5) 0 31. In 2003, sixty percef. of subscribers in Europe were men. Given that woman subscribers increase at the rate of 10 perc annt m and men at the rate of 5 percent per annum, what is the approximate '~:llliilll~l:i ibers between 2003 and 2010 in Europe? The subscription prices are volatile (4) 84 (5) 50 ...-..... the annual percent change in the gap between subscription revenues in the US and Europe. ;m,._...,,t he year in which the absolute value ofthis change is the highest? (1) 03-04 (2) 05-06 (3) 06-07 (4) 08-09 (5) 09-10
  • 10. w w w .exam race.com There are 100 employees in an organization across five departments. The following table gives the departement-wise distribution of average age, average basic pay and allowances. The gross pay of an employee is the sum of his/her basic pay and allowances. lh·partrncnt' fumbct- or Average ,vcrag{· Basic Allowances ~:mployees Age (Yl~an) !•ay (Rs. ) (% of Basic J•a.., IIR 5 5000 ()()!){) Finance 30 6500 Bu~incs~ DeveliJf.}ll'lent 42 Maimcnancc iO J5 a department w1th h1gher average age to ~em 1th lower average age. Questions below are independent : £f e 1-MJU 33. There was a mutual transfer of e between Market1ng and F1nance departments and transfer of one empioyee from M arke~!'' 1>s a result, the average age of F1nance department 1ncreased by one year and that of marke~rtm ent remained the same. What is the new average age of HR department? •(1) 30 (2) 35 (4) 45 (5) cannot be determined transfer a - person with basic pay of Rs. 8000 from the Marketing department? (1)~~)11% (3)13% (4)15% (5)17% ~mployees(each with a basic pay of Rs. 6000) are transferred from Maintenance department to HR department and one person (with a basic pay of Rs. 8000) was transferred from Marketing department to HR department, what will be the percemage change in average basic pay of HR departmem? (1) 10.5% (2) 12.5% (3) 15% (4) 30% (5)40%
  • 11. w w w .exam race.com Abdul, Bikram and Chetan are three professianal traders who trade in shares of a company XYZ Ltd. Abdul follows the strategy of buying at the opening of the day at 10 am and selling the whole Iot at the close of the day at 3 pm. Bikram follows the strategy of buying at hourly intervals: 10 am, 11 am, 12 noon, 1 pm and 2 pm, and sellingthe whole lot at the close of the day. Further, he buys an equal number of shares in each purchase. Chetan follows a similar pattern as Bikram but his strategy is somewhat different. Chetan's total investment amaunt is divided equally among his purchases. The profit or loss made by each investor is the difference between the sale value at the close of the day less the investment in purchase. The "returri' for each investor is defined as the ratio of the profit or loss to the investment amaunt e d as a percentage. 36. on a "boom" day the price of xvz Ltd. keeps rising throughout the day and peaks 0day. Which trader got the minimum return on that day? (1) Bikram (2) Chetan (3) Abdul (4) Abdul or Chetan (5) cannot be dG d 37. On a day of fluctuating market prices, the share price of XYZ Ltd. e~a Q in, i.e., it is higher at the close of the day compared to the opening value. Which trader got th~m return on that day? (1) Bikram (2) Chetan (3) Abdul (4) Bikram or Chetan G ot be determined 38. Which one ofthe following statements is always true? f'l:j- (1) Abdul will not be the one with the minimum retur .t.,(2) Return for Chetan will be higherthan that of Bikr m ~ (3) Return for Bikram will be higherthan that of e (4) Return for Chetan cannot be higher than tha Ab (5) none of the above ~ One day, two other traders, Dane~n 11 oi tl Abdul, Bikram and Chetan for trading in the shares of XYZ Ltd. Dane followed a strategy · qual numbers of shares at 10 am, 11 am and 12 noon, and selling the same numbers at21, and 3 pm. Emily, on the other hand, followed the strategy of buying shares using all her m a 0 am and selling all of them at 12 noon and again buyingthe shares for all the money at 1 p and & · selling all of them at the close of the day at 3 pm. At the close of the b ve~ (1) Share price was at its lowest at 2 pm (2) Share price was at its lowest at 11 am (3) Share price at 1 pm was higher than the share price at 2 pm (4) Share price at 1 pm was higher than the share price at 12 noon (5) none of the above
  • 12. w w w .exam race.com ~u. ~ll ell ~ IJI n..~ VV d :> dl. n :> 111 ~11~:>1. en (1) 10 am (2) 11 am (3) 12 noon (4) 1 pm (5) cannot be determined
  • 13. w w w .exam race.com (i) There arethree houses on each side ofthe road. (ii) These six houses are labeled as P, Q, R, S, T and U. (iii) The houses are of different colours, namely, Red, Blue, Green, Orange, Yellow and White. (iv) The houses are of different heights. (v) T, thetallest house, is exactIy opposite to the Red coloured house. (vi) The shortest house is exactIy opposite to the Green coloured house. (vii) U, the Orange coloured house, is located between P and S. (viii) (ix) (x) R, the Yellow coloured house, is exactly opposite toP. Q, the Green coloured house, is exactIy opposite to U. P, the White coloured house, is tallerthan R, but shorter than Sand Q. 41. What is the colour of the tallest house? r ~ (1) Red (2) Blue (3) Green (4)Yellow (S) none of these ~ 42. What is the colour of the house diagonally opposite to theYellow col0 st (1) White (2) Blue (3) Green (4) Red (S) none of these CJ ~'(j 43. Which is the second tallest house? (1) p (2) s (3)Q (4) R
  • 14. w w w .exam race.com In a sports event, six teams (A, B, C, D, E and F) are competing against each other. Matches are scheduled in two stages. Each team plays three matches in Stage-1 and two matches in Stage-11. No team plays against the same team morethan once in the event. No ties are permitted in any of thematches. The observations after the completion of Stage-I and Stage-II are as given below. Stage-1: • Oneteam won all the three matches. • Two teams lost all thematches. • D lost to Abut won against Cand F. • E lost to Bbut won against Cand F. • BIost at least one match. • Fdid not play against thetop team of Stage-1. Stage-11: • The leader of Stage-llost the next two matches. ~ + • Of the two teams at the bottom after Stage-1, oneteam wo at es, wh1le the other Iost both matches. • One moreteam lost both matches 1n Stage-11. f'l::f- 44. The team(s) w1th the most w1ns 1n the event 1s (ar · ~ (l)A (2)A&C (3)F (4)E & (l)A D &F (3) E& F (4) D, E& F (S) D& F
  • 15. w w w .exam race.com Telecom operators get revenue from transfer of data and voice. Average revenue received from transfer of each unit of data is known as ARDT. In the diagram below, the revenue received from data transfer as percentage of total revenue received and the ARDT in US Dollars (USD) are given for various countries. • •fJ-oi~flii'WI: {~:l,!:;t.~} $5 e ' he same in both countries. vent,_ in India is about 2times that of Singapore nue in India is about 4times that of Singapore revenue in Singapore is about 2 times that of India (S) Total revenue in Singapore is about 4 times that of India •Ja~n (lD13.10~) UK $Hi (3) Germany and USA
  • 16. w w w .exam race.com I Ul I I IUIQ Ql IU UUUUI o; I Ul .JYY <;U<;I lo ............UI I I<; 0.1 IQO. II I ~ U.I.UJ 0.1 I<; O.UO.Q I I o; Y<;l IU<; II I I I IUIQ I'"' 0. YY 1'-'<; 0.1 IQO. Ul ,J YY<;U<;I I and that the volume of data transfer is the same in both the countries. What is the percentage increase of ARDT in India if there is no change in ARDT in Sweden? (1) 400% (2) 550% (3) 800% (4) 950% (5) cannot be determined
  • 17. w w w .exam race.com This section contains 40 questions Directions for questions 51 to 54: In each question, there are five sentences. Each sentence has a pair of words that are itaIicized and highlighted. From the italicized and highlighted words, select the most appropriate words (A or B)to form correct sentences. The sentences arefollowed by optionsthat indicate the words, which may be selected to correctly compietethe set of sentences. From the options g1ven, choose the mostappropriate one. 51. An1ta wore a beaut1ful broach {A}/brooch{B} on the lapel of herJacket. coundlfol(A}/counsefoi(B}. CJI wauld like youradvke(A}/advise{B} on which job I should choose. The last scene provided a climactic(A)/climatic{B) ending to the film. Jeans thatjiair{A}/ffore{B} at the bottom are in fashion these days. + (1) BABM (2) BABAB (3) BAMB (4)ABABA (1) BBABA (2) BBAM (5) BABAB 53. ke+Nay as thetrain elimbed the hiII. (3) BAABB (4)ABBM (5) BBBBA The hockey team cauld not deal with his unsociable{A}/unsociai{B} tendencies. (1) BABBA (2) BBABB (3)BABM (4)ABBAB (5)AABBA
  • 18. w w w .exam race.com usage (including spelling punctuation and logical consistency). Then, choose the most appropriate option. 55. A. In 1849, a poor Bavarian imigrant named Levi Strauss B. landed in San Francisco, california, C. at the invitation of his brother-in-law David Stern D. owner of dry goods business. E. Th1s dry goods busmess would later become known as Lev1 Strauss & Company. (1) B only (2) Band C (3)AandB (4) A only (S)A, B andD CJ 56. A. In response to the allegations and condemnation pouring in, B. Nike implemented comprehensive changes in their labour policy. C. Perhaps sensingthe rising tide of global labour concerns, ~., + D. from the public wauld become a prominent media issue, " - / E. Nike sought to be a industry leader in employee relations. r ~ (1) D and E (2) D only (3)Aand E (4)Aand D~~dE 57. .t"'V A. Charges and counter charges mean nothing ~ B. to the few miIlion who have lost their hom D. is still unableto reach hundreds wh re o ed. E. The death count haveJUSt be::.t..=__C (1)Aonly (2)Conly ~~:v~ (4)A,CandD 58. to ma~ r you. (S) D only B. Because you'd Inett I associate you more with my parents than with me. ·~~ C. And yet yo"'"""'-.., my cousins in Calcutta, who seem so innocent and obedient when I visited them. urious about me in the least. u did make effort to meet me. (2)AandB (3)AandE (4) D only (S)Aand D
  • 19. w w w .exam race.com Given below each question arefive pairs of words. Choosethe pairthat best compietes the sentence. 59. The genocides in Bosnia and Rwanda, apart from being mis-described in the most sinister and ___ manner as 'ethnic cleansing', were also blamed, in further hand-washing rhetoric, on something dark and interiorto _____ and perpetrators alike. (1) 1nnovat1ve; commumcator (2) enchant1ng; leaders (3) d1s1ngenuous; v1ct1ms (4) ex1gent; explo1ters (5) trag1c; sufferers contrary, anc1ent astronomers wereforced to _____ that certa1n bod1es ml~=~1n c1rcles about po1nts, wh1ch 1n turn moved 1n c1rcles about the earth. ~ (1) scrut1n1zers; believe (4) observers; concede ~.. + (2) observers; agree (5) students; con~~"'fJ (3) scrut1n1zers; suggest .:'-/ 61. £~Every human be1ng, after the fwst few days of h1s hfe, 1 :U'twofactors; on the one hand, there :::'s-=--=--.___endowment; and on the~ ere 1s the effect of env1ronment, 1nclud1ng (1) const1tut1onal; weather ~~~nom1c;learn1ng (2) congenital; educat1on ;f )genet1c; pedagogy (3) personal; climate 62. (kExhaustion of natural resou , d ruct1on of 1nd1v1dual 1n1t1at1ve by governments, control overmen's mmd> by'"'"~ "'"""ood propogoodo •~ ~'"""h• '"'I"~"' wh'oh •PP~' <o be on the 1ncreas ul~f the 1mpact of sc1ence upon m1nds su1ted by ____to an earher kmd of world. (1) tenet n (4) organs; trad1t1on (2) a s· h1 1t1ons (5) departments; repuls1on
  • 20. w w w .exam race.com incorrect or inappropriate. 63. Run (1) I must run fast to catch up with him. (2) Our team scored a goal against the run of play. (3) You can't run over him like that. (4) The newly released book is enjoying a popular run. (5) This film is a run-of-the-mill production. 64. Round (1) The policefired a round of tear gas shells. (2) The shop is located round the corner. (3) Wetook a ride on the merry-go-round. (4) The doctor is on a hospital round. ~• + (5) I shall proceed further only after you come around to admitting it. " - / 65.Buckle CJ(1) .after the long hike our knees were beginning to buckle. ~ (2) The horse suddenly broke into a buckle. .t"'V (3) The accused did not buckle under police interr ·~ (4) Sametimes, an earthquake can make a brid ,..... ckl . (5) People should Iearn to buckle up as soon ey 66.File ~ (1) You will find the paper in the~~rV (2) I need to file an insuran~· . (3)The cadets were marchi i s gl file. (4) File your na 1 i~~:~~~u ail polish. (5) When the p~~n-+soldier brokethefile.
  • 21. w w w .exam race.com in the mostappropriate way. 67. Most people at their first consultation take a furtive look at the surgeon's hands in the hope of reassurance. Prospective patients look for delicacy, sensitivity, steadiness, perhaps unblemished pallor. On this basis, Henry Perowne loses a number of cases each year. Generally, he knows it's about to happen before the patient does: the downward glance repeated, the prepared questions beginning to falter, the overemphat1c thanks dunng the retreat to the door. (1) Other people do not commumcate due to thew poor observation. (2) (3) But Perowne h1mself 1s not concerned. (4) But others will take the1r place, he thought. r ~ (5) These hands are steady enough, but they are large. ~ 68. Trade protectionism, disguised as concern for the elimate, is raisinl!l..liliw·~ concerns, powerful industrialized countries are holding out threat of a I other. (3) Jews in India were always tolerant. (4) Reiigious tolerance has always been only afa9ade and nothing more. (5) The pretty pastel streets are, thus, very popular with thetourists. •ng competitiveness n imports of energy·
  • 22. w w w .exam race.com 70. Given the cultural and intellectual interconnections, the question of what is 'Western' and what is 'Eastern' (or Indian) is often hard to decide, and the issue can be discussed only in more dialectical terms. The diagnosis of athought as 'purely Western' or 'purely Indian' can be very illusory. (1) Thoughts are not the kind of things that can be easily categorized. (2) Though 'accidentalism' and 'orientalism' as dichotomous concepts have found many adherents. (3) 'East is East and West is West' has been a discredited notion for a longtime now. (4) Compartmentalizingthoughts is often desirable. (5) The origin of athought is not the kind of thingto which 'purity' happens easily. CJ ,Jo· &'(j 0~ • ~
  • 23. w w w .exam race.com "-'IIUU;)t:: U It:: Ill U;)l dJ-'J-'1 Ut-JII dlt:: dll;) YYt::l lU t::d'-'11 yut::;)UUIIo Language is not a cultural artifact that we learn the way we learn to tell time or how the federal government works. Instead, it is a distinct piece of the biological makeup of our brains. Language is a complex, specialized skill, which develops in the child spontaneously, without conscious effort or formal instruction, is deployed without awareness of its underlying Iogic, is qualitatively the same in every individual, and is distinct from more general abilities to process information or behave intelligently. For these reasons same cognitive scientists have described language as a psychological faculty, a mental organ, a neural system, and a computational module. But I prefer the admittedly quaint term "instinct." It conveys the idea that people know how to talk in more or Iess the sensethat spiders know ho webs. Web-spinning was not invented by same unsung spider genius and does not depend onlr:w~!l the right education or on having an aptitude for architecture or the construction trades. Ra r, s rs spin spider webs because they have spider brains, which give them the urge to spin aa etence to succeed. Although there are differences between webs and words, I will enc age u to see Ianguage in this way, for it helps to make sense of the phenomena we wiII explorr r ~ Thinking of language as an instinct inverts the popular wisdom, especially as it h~passed down in the canon of the humanities and social sciences. Language is no more as i'Vntion than is upright posture. It is not a manifestation of a general capacity to use -symbols: t ear old, we shall see, is a grammatical genius, but is quite incompetent at the visual arts, a graphy, traffic signs, and the other staples of the semiotics curriculum. Though language i ma ificent ability unique to Homo sapiens among living species, it does not call for sequester~ ' of humans from the domain of biology, for a magnificent ability unique to a particular i' ·_s is far from unique in the animal kingdom. Same kinds of bats home in on flying inse usi pier sonar. Some kinds of migratory birds navigate thousands of miles by calibrating t o ·ons of the constellations against the time of day and year. In nature's talent show we are si y a e s of primate with our own act, a knack for communicating information about who did w. t t ho by modulating the sounds we make when we exhale Once you begin to Ioak at language adaption to communicate inforllloll~J!~ of thought, and, we shall se~a· · ot oreover, seeing Ianguage as one of nature's engineering marvels - an organ with "that perf · ructure and co-adaption which justly excites our admiration, "in Darwin's words- a ·spect for your ordinary Joe and the much-maligned English language (or any language). of language, from the scientist's point of view, is part of our biological .,."-iru> that parents teach their chiIdren or samething that must be elaborated in ~~e said, "Education is an admirable thing, but it is well to remember from time to is worth knowing can be taught." A preschooler's tacit knowledge of grammar is more'.ff't:q ~c:~!d than the thickest style manual or the most state-of-the-art computer language same applies to all healthy human beings, even the notorious syntax-fracturing athlete and the, you know, Iike, inarticulate teenage skateboarder. Finally, since Ianguage is ~~duct of a well-engineered biological instinct, we shall see that it is not nutty barrel of monkeys that entertainer-columnists make it out to be. 71. Accordingto the passage, which ofthe following does not stem from popular wisdom on language? (1) Language is a cultural artifact. (2) Language is a cultural invention. (3) Language is learnt as we grow. (4) Language is unique to Homo sapiens
  • 24. w w w .exam race.com 72. Which of the following can be used to repIacethe "spiders know how to spin webs" analogy as used by the author? (1) A kitten learningto jump over a wall (2) Bees collecting nectar (3) A donkey carrying a load (4) A horse running a Derby (5) A pet dog protecting its owner's property 73. Accordmgto the passage, wh1ch ofthe follow1ng 1s umque to human bemgs? (1) Ability to use symbols while communicating with one another. (2) Ability to communicate with each other through voice modulation. (3) Ability to communicate informationto other members of the species. CJ(4) Ability to use sound as means of communication. (5) All ofthe above. 74. According to the passage, complexity of language cannot u0p!entsor at school to ch1ldren because (1) children 1nSt1nct1vely know language. flj(2) children learn the language on the1r own. ~ (3) language 1s not amenable to teachmg. (4) ch1ldren know language better than the1r teac r ents. (5) ch1ldren are born w1th the knowledge of s cs.
  • 25. w w w .exam race.com "-'IIUU;)t:: U It:: IIIU;)l dJ-'J-'1 Ut-JII dlt:: dll;) YYt::l lU t::d'-'11 yut::;)UUIIo When I was little, children were bought two kinds of ice cream, sold from those white wagons with the canopies made of silvery metal: either the two-cent cone or the four-cent ice cream pie. The two-cent cone was very smaII, in fact it cauld fit comfortably into a child's hand, and it was made by taking the ice cream from its container with a special scoop and piIing it on the cone. Granny always suggested I eat only a part of the cone, then throw away the pointed end, because it had been touched by the vendor's thing, the biscuit surfaces softening as they became soaked in creamy nectar. give here: in theory the pies had been touched only by the machine; in practi them against his hand while giving them to us, but it was impossible to i~e the. aminated area. 1was fascinated, however, by same ofmy peers, whose parents bas m t afour-cent pie but two two-cent cones. These privileged children advanced proudly with ne c their right hand and one in their left; and expertly moving their head from side to sidr & first one, then the other. This Iiturgy seemed to me so sumptuously enviable, that man ed to be allowed to eelebrate it. In vain. My elders were inflexible: a four-cent ice, yes; bu o t - nt ones, absolutely no. assuming that in due course the tips of es ere discarded. The pathetic, and obviously mendacious, justification was that a boy~- e WI h turning his eyes from one coneto the other was more inelined to stumble over stones, e , acks in the pavement. I dimly sensed that there was another secretjustification, cruelly p ag · , but I was unable to grasp it. Today, citizen and victim of!a r society, a civilization of excess and waste (which the society of the thirties was not), I reali t se dear and now departed elders were right. Two two-cent cones instead of one at fou cents t signify squandering, economically speaking, but symbolically they surely did. It was ·s pr. ise reason, that I yearned for them: because two ice creams suggested ·sely why they were denied me: because they Iooked indecent, an insult to poverty, a di ;;,llllo;.'l....,.itious privilege, a boast of wealth. Only spoiled chiIdren ate two cones at once, those chil9il,._m!o in fairy tales were rightly punished, as Pinocchio was when he rejected the skin and h parents who encouraged this weakness, appropriate to Iittle parvenus, were bringing up re 1n the foolish theater of "I'd like to but I can't." They were preparing them to turn up at s cheek-in with a fake Gucci bag bought from a street peddler on the beach at Rimini Nowadays the moralist risks seeming at odds with morality, in a world where the consumer civilization now wants even adults to be spoiled, and promises them always something more, from the wristwatch in the box of detergent to the bonus bangle sheathed, with the magazine it aceompanies, in a plastic envelope. Like the parents of those ambidextrous gluttons I so envied, the consumer civiiization pretends to give more, but actually gives, for four cents, what is worth four cents. You wiII throw away the old transistar radio to purchase the new one, that boasts an alarm clock as well, but same
  • 26. w w w .exam race.com not Iast nearly so long as the glorious old Fiat 500, which, even when it broke down, could be started again with a kick. The morality of the old days made Spartans of us all, while today's morality wants all of us to be Sybarites. 76. Which of the following cannot be inferred from the passage? (1) Today's society is more extravagant than the society of the 1930s. (2) The act of eatingtwo ice cream cones is akin to a ceremonial process. (3) Elders rightly suggested that a boy turning eyes from one cone to the other was more li (4) Despite seeming to promise more, the consumer civilization gives away exactlyO ' worth. (5) The consumer civilization attempts to spoil chiIdren and adults alike. CJ 77. In the passage, the phrase "Iittle parvenus" refers to 0(1) naughty midgets. + (2) old hags. CJ(3) arrogant people. (4) young upstarts. ~ (5) foolish kids. "'v (1) it made dietetic sense. (2) it suggested intemperance. flj(3) it was more fun. (4) it had a visual appeal. (5) he was a glutton. 79. What does the author bl "r'iclW<Idays the moralist risks seeming at odds with morality"? (1) The moralist of ves>ter·dav,_lllllll~·eccrme immoral today. (3) dialectic. (4) diatonic. (5) diastolic. 1+s changed overthe years. ~,jO'Ci~~ with immorality have gone up. of morality is fast becoming popular. to the author, the justification for refusal to let him eat two cones was piausibly
  • 27. w w w .exam race.com "-'IIUU;)t:: U It:: IIIU;)l dJ-'J-'1 Ut-JII dlt:: dll;) YYt::l lU t::d'-'11 yut::;)UUIIo A remarkable aspect of art of the present century is the range of concepts and ideologies which it embodies. It is aimost tempting to see a pattern emerging within the art field - or alternative!y imposed upon it a posteriori - similar to that which exists under the umbrella of science where the general term covers a whole range of separate, though interconnecting activities. Any parallelism is however- in this instance at least - misleading. A scientific discipiine develops systematically once its bare tenets have been established, named and categorized as conventions. Many of the concepts of modern art, by contrast, have resulted from the aimost accidental meetings of groups of talented individuals a'fi""J! times and certain places. The ideas generated by these chance meetings had twofold conc."""''" Firstly, a corpus of work wauld be produced which, in great part, remains as a concrete r rd he events. Secondly, the ideas wauld themselves be disseminated through many di~ els of communication - seeds that often bore fruit in contexts far removed from ta · ra · n. Not all movements were exclusively concerned with innovation. Surrealism, for instan clai o embody a kind of insight which can be present in the art of any period. This claim has bee " ally accepted so that a sixteenth century painting by Spranger or a mysterious photogri ! t ~ can Iegitimately be discussed in surrealist terms. Briefly, then, the concepts of modern re f many different (often fundamentally different) kinds and resulted from the exposures fJ. ""rs, sculptors and thinkers to the more complex phenomena of the twentieth century, i I o ever increasing knowledge of the thought and products of earlier centuries. Different gr p a sts would collaborate in trying to make sense of rapidly changing world of visual and spi~· I e er. nee. We should hardly be surprised if no one group succeeded completely, but achiev e < through relative, have been considerable. Landmarks have been established - concrete st me ' osition which give a pattern to a situation which could easily have degenerated into al os. Beyond this, new language tools have been created for those who follow - seml ] i ems wh1ch can prov1de a spnngboard for further explorations. The cod1fy1ng of art 1s often cnt1~ y one can understand that art1sts are wary of be1ng pigeon- holed smce they are apt tar at mselves as 1nd1v1duals- somet1mes w1th good reason. The not1on of self-express1on, howeve o e carnes qu1te the we1ght 1t once d1d; obJectiVIty has 1ts defenders. There 1s good reason acce 1deas cod1f1ed by art1sts and cnt1cs, over the past s1xty years or so, as st s ~ndependent existence - an independence which is not without its own portant here. As an art movement slips into temporal perspective, it ceases - becoming, rather, afossil. This is not to say it becomes useless or uninteresting >'ili'"ollacan reconstruct the life of a prehistoric environment from the messages codified into the sttl~~~~f fossil, so can an artist decipher whole webs of intellectual and creative possibility from tructure of a 'dead' art movement. The artist can match the creative patterns crystallized ructure against the potentials and possibilities of his own time. AS T.S Eliot observed, no one nything from scratch; however consciously you may try to Iive in the present, you are still involved with a nexus of behaviour patterns bequeathed from the past. The original and creative person is not sameone who ignores these patterns, but someone who is able to translate and develop them so that they confirm more exactly to his- and our- present needs.
  • 28. w w w .exam race.com (2)the dissemination of ideas through the state and its organizations. (3)accidental interactions among people blessed with creative muse. (4)patronage by the rich and powerful that supported art. (5)systematic investigation, codification and conventions. 82. In the passage, the word 'fossil' can be interpreted as (1) an art movement that has ceased to remain interesting or useful. (2)an analogy from the physical world to indicate a historic art movement. (3)an analogy from the physical worldto indicatethe barrenness of artistic creations in th (4)an embedded codification of pre-historic life. e(5)an analogy from the physical world to indicatethe passing of an era associated wit an a movement. r ~ 83. In the passage, which of the following similarities between science and art~ad to erroneous conclusions? ~.. + (l)Both, in general, include a gamut of distinct but interconnect~n ~ (2)Both have movements not necessarily concerned with innov n. (3)Both depend on collaborations between talented indivil l j (4)Both involve abstract thought and dissemination o~f.· (5)Both reflect compiex priorities of themodern worl~ 84. The range of concepts and ideologies emb in art of the twentieth century is explained by (l)the existence of movements such as surr (2)1andmarks which give a pattern tot only thrive in a 'greenhouse' insulated from the past biases. s' and 'original thinking' interpret and develop on past thoughts to suit contemporary
  • 29. w w w .exam race.com "-'IIUU;)t:: U It:: IIIU;)l dJ-'J-'1 Ut-JII dlt:: dll;) YYt::l lU t::d'-'11 yut::;)UUIIo To summarize the Classic Maya collapse, we can tentatively identify five strands. I acknowledge, however, that Maya archaeologists still disagree vigorously among themselves-in part, because the different strands evidently varied in importance among different parts of the Maya realm; because detailed archaeological studies are available for only same Maya sites; and because it remains puzzling why most of the Maya heartIand remained nearly empty of population and failed to recover after the collapse and after re-growth of forests. With those caveats, it appears to me that one strand consisted of population growth o available resources: a dilemma similar to the one foreseen by Thomas Malthus in 1798 and out today in Rwanda (Chapter 10), Haiti (Chapter 11), and elsewhere. 1>s the arch o Webster succinctly puts it, "Too many farmers grew too many crops on too much Compounding that mismatch between population and resources was the second deforestation and hillside erosion, which caused a decrease in the amount of us when more rather than less farmIand was needed, and possibly exacerbate by drought resulting from deforestation, by soil nutrient depletion and other soiI p struggleto prevent bracken ferns from overrunningthe fields. + The third strand consisted of increased fighting, as more and mo pea ght over fewer resources. Maya warfare, already endemic, peaked just beforethe coli~. 1 tis t surprising when one reflects that at Ieast 5,000,000 people, perhaps many more, were o an area smaller than the state of Colorado (104,000 square miles). That warfare w~ eased further the amount of Iand available for agriculture, by creating no-man's lands ee ncipalities where it was now unsafe to farm. Bringing matters to a head was the strand of t ange. The drought at the time of the Classic collapse was not the first drought that the Ma ad ' through, but it was the most severe. tit the time of previous droughts, there were still u ab1 parts of the Maya Iandscape, and people at a site affected by drought could save thems~ oving to another site. However, by the time of the Classic collapse the landscape was now II as no useful unoccupied land in the vicinity on which to begin anew, and the whoIe po5 :ul ion u not be accommodated in the few areas that continued to have reliable water supplies. 1>s our fifth strand, we ha..Qfln er why the kings and nobles failed to recognize and solve these seemingly obvious problem~ining their society. Their attention was evidently focused on their short-term concerns enricbing themselves, waging wars, erecting monuments, competing with each other, and extra 1 ugfrfood from the peasants to support all those activities. Like most leaders throughout h r , , the Maya kings and nobles did not heed long-term problems, insofar as they perceived he II return to this theme in Chapter 14. w stiII have same other past societies to consider in this book before we switch our to e modern world, we must already be struck by some parallels between the Maya and the ies discussed in Chapters 2-4. As on Easter Island, Mangareva, and among the Anasazi, Maya • <it:tnmental and population problems led to increasing warfare and civil strife. As on Easter Island and co Canyon, Maya peak population numbers were followed swiftIy by political and social coliapse. ParaIIelingthe eventual extension of agriculture from Easter Island's coastal Iowlands to its uplands, and from the Mimbres floodplain to the hills, COpan's inhabitants also expanded from the floodplain to the more fragile hill slopes, leaving them with a larger population to feed when the agricultural boom in the hills went bust. Like Easter Island chiefs erecting ever larger statues, eventually crowned by pukao, and Iike Anasazi elite treatingthemselves to necklaces of 2,000 turquoise beads, Maya kings sought to outdo each other with more and more impressive tempies, covered with thicker and thicker piaster- reminiscent in turn of the extravagant conspicuous
  • 30. w w w .exam race.com consumption by modern American CEOs. The passivity of Easter chiefs and Maya kings in the face of the real big threats to their societies completes our list of disquieting parallels. 86. Accordingto the passage, which of the following best represents the factor that has been cited by the author in the context of Rwanda and Haiti? (1) Various ethnic groups competingfor land and other resources (2) Various ethnic groups competingfor limited land resources (3) Various ethnic groups fighting wit each other (4) Various ethnic groups competingfor political power (5) Various ethnic groups fighting for their identity 87. By an anthropogenic drought, the author means CJ(1) A drought caused by lack of rains. (2) A drought caused due to deforestation ~.., + (3) A drought caused by failure to prevent brackenferns from oveu~lields. (4) A drought caused by actions of human beings. (5) A drought caused by elimate changes. ~ 88. Accordingto the passage, the drought at thetime ~;u:;sehad a different impact compared to the droughts earlier because ~ (1)The Maya kings continue to be extravagant§ com on people were suffering. (2) It happened at thetime of collapse ~ol a <hip • ong Mayas. (3) It happened when the Maya popula n upied all available land suited for agriculture. (4) It was foilowed by internecina :w ar m g Mayans. (5) Irreversible environmental d · d to this drought. 89. Accordingto the autho.a#t ifficult to explain the reasons for Maya collapse? (1) COpan inhabitan de~~records of that period. (2) tat• n and hillside erosion have wiped out all traces of the Maya kingdom. (3) Mayas do not provide any consistent evidence. (4) sibleto ascertain which ofthe factors best explains as to why theMaya llapsed. 1"':~~ factor has not been cited as one of the factors causingthe collapse of Maya society? vironmental degradation dueto excess population 5ocial collapse dueto excess population Increased warfare among Maya people dimate change Obsession of Maya population with their own short-term concerns.